73 Best Upsc Practice Questions - March 2022

You might also like

Download as pdf or txt
Download as pdf or txt
You are on page 1of 159

Click Here To Watch The

Video
CURRENT AFFAIRS MCQ SERIES
MARCH 2022 CA
TARGET PRELIMS 2023
Q.1) According to the Constitution, which of the
PYQ ASKED ON SIMILAR LINE
following fundamental rights are available to both
foreigners and citizens?
Q. Which Article of the Constitution
1. Equality before law and equal protection of laws. of India safeguards one’s right to
2. Movement, residence and profession. marry the person of one’s choice?
3. Right to elementary education. [2019]
4. Protection of language, script and their culture. (a) Article 19
(b) Article 21
Select the correct answer using codes given below: (c) Article 25
1. 1, 2 and 3 only
(d) Article 29
2. 1 and 3 only
3. 2, 3 and 4 only
4. 2 and 4 only
Ans: B

∙ Fundamental rights are certain claims which are fundamental to the all-round development of an individual. These rights
are recognized by the state and guaranteed by the Constitution itself. These claims are termed as Fundamental Rights
(FRs). Fundamental Rights prevent arbitrary rule of the state by promoting the ideal of political democracy,
∙ Fundamental rights available to both citizens and foreigners except enemy aliens
∙ Article 14 - Equality before the law and equal protection of laws.
∙ Article 20 - pertains to Protection in respect of conviction for offences.
∙ Article 21 - signifies Protection of life and personal liberty.
∙ Article 21A - Right to elementary education.
∙ Article 22 - Protection against arrest and detention in certain cases.
∙ Article 23 - Prohibition of traffic in human beings and forced labour.
∙ Article 24 - Prohibition of employment of children in factories etc.
∙ Article 25 - Freedom of conscience and free profession, practice and propagation of religion.
∙ Article 26 - Freedom to manage religious affairs.
∙ Article 27 – Freedom from payment of taxes for promotion of any religion.
∙ Article 28 – Freedom from attending religious instruction or worship in certain educational institutions.
∙ There are some Fundamental rights which are only available to citizens of India are:
∙ Article 15: Right against discrimination on the grounds of religion, race, caste, sex or place of birth;
∙ Article 16: right to equality of opportunity in matter of public employment;
∙ Article 19: freedom of speech and expression, assembly, association, movement, residence and profession;
∙ Article 29 and 30: cultural and educational rights.
Q.2) Consider the following statements regarding the
PYQ ASKED ON SIMILAR LINE
section 144 of Criminal Procedure Code of 1973:
Q. With reference to India, consider the following
1. The maximum punishment for violation of section 144 of statements: [2021]
CrPC is one year. 1.Judicial custody means an accused is in the
2. Section 144 is imposed in urgent cases that have the custody of the concerned magistrate and such
potential to cause trouble or damage to human life or accused is locked up in police station, not in jail.
property. 2.During judicial custody, the police officer in
charge of the case is not allowed to interrogate the
3. Curfew is different from section 144, as it orders people
suspect without the approval of the court.
to stay indoors for a specific period of time.
Which of the statements given above is/are
Which of the statements given above is/are correct? correct?
a) 1 and 2 only
b) 2 and 3 only (a) 1 only
c) 1, 2 and 3 (b) 2 only
d) 1 and 3 only (c) Both 1 and 2
(d) Neither 1 nor 2
Ans: B

∙ Statement 1 is incorrect: Section 144 of the Criminal Procedure Code (CrPC) of 1973 generally prohibits public gathering.
It authorises the Executive Magistrate of any state or territory to issue an order to prohibit the assembly of four or more
people in an area.According to the law, every member of such 'unlawful assembly' can be supense for engaging in rioting.
The maximum punishment for violation of section 144 is three years. Moreover, obstructing police from breaking up an
unlawful assembly is a punishable offence as well. Section 144 also restricts carrying any sort of weapon in that area
where section 144 has been imposed and people can be detained for violating it.
∙ Statement 2 is correct: Section 144 is imposed in urgent cases of nuisance or apprehended danger of some event that
has the potential to cause trouble or damage to human life or property. No order under Section 144 shall remain in force
for more than two months but the state government can extend the validity for two months and maximum up to six
months. It can be withdrawn at any point of time if the situation becomes normal.Section 144 was used for the first time
in 1861 by the British Raj, and thereafter became an important tool to stop all nationalist protests during the Freedom
Struggle. However, the use of the section in Independent India remains controversial as very little has changed.
∙ Statement 3 is correct: Section 144 is generally prohibitory in nature. It restricts from public gathering but doesn't bar it
all together. A curfew, on the other hand, orders people to stay indoors for a specific period of time. So, the authorities
can impose curfew for a certain period of time (However, the authorities can also extend the curfew if need be). One also
needs prior approval from the local police for moving out during curfew.
Q.3) With reference to National Commission for Scheduled Tribe,
consider the following statements:
PYQ ASKED ON SIMILAR LINE

1. Article 338A gives powers to the NCST to oversee the Q. Which of the following bodies does
implementation of various safeguards under any law provided to not/do not find mention in the
STs. Constitution? [2013]
2. The Constitution lays down that the composition and strength of 1. National Development Council
shall be determined by the Parliament. 2. Planning Commission
3. NCST presents a report to the President, annually and at such 3. Zonal Councils
other times as the Commission may deem fit. Select the correct answer using the codes
given below
Which of the statements given above is/are not correct?
(a) 1 and 2 only
a) 1 only (b) 2 only
b) 2 and 3 only (c) 1 and 3 only
c) 2 only (d) 1, 2 and 3 only
d) 1 and 3 only
Ans: C

∙ Statement 1 is correct: National Commission for Scheduled Tribe is a constitutional body that was set up with effect
from 19th February 2004 by amending Article 338 and by inserting a new article 338A in the Constitution through the
89th Constitution Amendment Act, 2003. Article 338A inter-alia gives powers to the NCST to oversee the implementation
of various safeguards provided to STs under the Constitution or under any other law for time being in force or under any
other order to the Government and to evaluate the working of such safeguards.
∙ Statement 2 is incorrect: The Constitution lays down the composition of NCST, consisting of a Chairperson, a
Vice-Chairperson and 3 other Members who are appointed by the President by warrant under his hand and seal.At least
one member should be a woman.The Chairperson, the Vice-Chairperson and the other Members hold office for a term of
3 years.The members are not eligible for appointment for more than two terms.The Chairperson has been given the rank
of Union Cabinet Ministers, the Vice Chairperson has the rank of a Minister of State and other Members have the rank of
a Secretary to the Government of India.
∙ Statement 3 is correct: One of the duties and functions performed by National Commission for Scheduled Tribe is present
to the President, annually and at such other times as the Commission may deem fit, reports upon the working of those
safeguards. Other functions like Commission inquires specific complaints with respect to the deprivation of rights and
safeguards of the STs and discharge such other functions in relation to the protection, welfare and development and
advancement of the Scheduled Tribes as the President may be subject to the provisions of any law made by Parliament by
rule specified.
Q.4) Consider the following statements regarding principles of
reasonable accommodations:
PYQ ASKED ON SIMILAR LINE
Q. Consider the following statements: [2019]
1. The principal provides additional support to persons with
1.As per recent amendment to the Indian Forest Act,
disabilities to facilitate their participation. 1927, forest dwellers have the right to fell the
2. Workers living with HIV and AIDS are not included in bamboos grown on forest areas.
categories of workers in the guideline on promoting diversity 2.As per the Scheduled Tribes and Other Traditional
and inclusion through workplace adjustments. Forest Dwellers (Recognition of Forest Rights) Act,
3. Rights of People with Disabilities Act, 2016, define 2006, bamboo is a minor forest produce.
‘discrimination’ in Section 2(h) includes ‘denial of reasonable 3.The Scheduled Tribes and Other Traditional Forest
accommodation’. Dwellers (Recognition of Forest Rights) Act, 2006
allows ownership of minor forest produce to forest
dwellers.
Which of the statements given above is/are correct? Which of the statements given above is/are correct?
(a) 1 and 2 only
a) 1, 2 and 3 (b) 2 and 3 only
b) 2 and 3 only (c) 3 only
c) 1 and 3 only (d) 1, 2 and 3
d) 1 and 2 only
Ans: C

∙ Statement 1 is correct: Reasonable accommodation’ is a principle that promotes equality, enables the grant of positive
rights and prevents discrimination based on disability, health condition or personal belief.Its use is primarily in the
disability rights sector. It captures the positive obligation of the State and private parties to provide additional support to
persons with disabilities to facilitate their full and effective participation in society.
∙ Statement 2 is incorrect: The International Labour Organisation in 2016, came out with a practical guide on promoting
diversity and inclusion through workplace adjustments. The need for workplace accommodation can arise in various
situations, but four categories of workers were chosen for the guide: Workers with disabilities,Workers living with HIV and
AIDS,Pregnant workers and those with family responsibilities, and Workers who hold a particular religion or belief.These
categories of workers come across different kinds of barriers at work. These can result either in loss of employment or in
lack of access to employment. The provision of reasonable accommodation plays a major role in addressing these barriers
and thus contributes to greater workplace equality, diversity and inclusion.
∙ Statement 3 is correct: In India, the Rights of People with Disabilities Act, 2016, defines ‘reasonable accommodation’ as
“necessary and appropriate modification and adjustments, without imposing a disproportionate or undue burden in a
particular case, to ensure to persons with disabilities the enjoyment or exercise of rights equally with others”. The Section
2(h) defines ‘discrimination’ as 'denial of reasonable accommodation’. Vikash Kumar v. UPSC (2021): The Supreme court
held that benchmark disability, that is a specified disability to the quota of 40 percent, is related only to special
reservation for the disabled in employment, but it should not be a restriction for other kinds of accommodation.
Q.5) Consider the following statements regarding Foreigners Tribunal:

1. Foreigners Tribunal is a quasi- Judicial body formed under the Foreigners’ Act, 1946.
2. All States and Union Territories can set up tribunals to decide whether a person staying
illegally in India is a foreigner or not.
3. Only the State administration can move the Tribunal against a suspect.

Which of the statements given above is/are correct?

a) 1 and 2 only
b) 1 and 3 only
c) 2 and 3 only
d) 1, 2 and 3
Ans: A

∙ Statement 1 is correct: Foreigners’ Tribunals are quasi-judicial bodies established as per the Foreigners’ Tribunal Order,
1964 and the Foreigners’ Act, 1946. Composition of Foreigners Tribunal includes Advocates not below the age of 35 years
of age with at least 7 years of practice (or) Retired Judicial Officers from the Assam Judicial Service (or) Retired IAS of ACS
Officers (not below the rank of Secretary/Addl. Secretary) having experience in quasi-judicial works.
∙ Statement 2 is correct: The Ministry of Home Affairs (MHA) has amended the Foreigners (Tribunals) Order, 1964, and
has empowered district magistrates in all States and Union Territories to set up tribunals (quasi-judicial bodies) to decide
whether a person staying illegally in India is a foreigner or not. Earlier, the powers to constitute tribunals were vested only
with the Centre.
∙ Statement 3 is incorrect: The amended order (Foreigners (Tribunal) Order, 2019) also empowers individuals to approach
the Tribunals. Earlier, only the State administration could move the Tribunal against a suspect. A declared foreigner, or
DF, is a person marked by Foreigners’ Tribunal (FT) for allegedly failing to prove their citizenship after the State police’s
Border wing marks him or her as an illegal immigrant.
Q.6) Recently Legal Aid Clinic was in the news is PYQ ASKED ON SIMILAR LINE
launched by:
Q. In India, Legal Services Authorities provide free
legal services to which of the following type of
a) World Health Organisation citizens?
b) National Commission for Women 1.Person with an annual income of less than Rs.
1,00,000
c) Delhi State Legal Services Authority 2.Transgender with an annual income of less than Rs.
d) Both (B) and (C) 2,00,000
3.Member of Other Backward Classes (OBC) with an
annual income of less than Rs. 3,00,000
4. All Senior Citizens
Select the correct answer using the code given below:
(a) 1 and 2 only
(b) 3 and 4 only
(c) 2 and 3 only
(d) 1 and 4 only
Ans: D

∙ Recently, the National Commission for Women (NCW) in collaboration with Delhi State Legal Services Authority (DSLSA)
has launched a Legal Aid Clinic. It will act as a single-window facility for resolving the grievances of women by offering
them free legal assistance.The legal aid clinic will operate out of the NCW office in New Delhi. Under the clinic,
counselling will be provided for walk-in complainants, women in distress, advice and information on various schemes of
the National Legal Services Authority (NALSA), free legal aid, hearings in matrimonial cases and other complaints
registered with the Commission.
∙ National Commission for Women:It was set up as a statutory body in January 1992 under the National Commission for
Women Act, 1990. Its mission is to strive towards enabling women to achieve equality and equal participation in all
spheres of life by securing her due rights and entitlements through suitable policy formulation, legislative measures, etc.
∙ Functions of the National commission for women
∙ Review the constitutional and legal safeguards for women.
∙ Recommend remedial legislative measures.
∙ Facilitate redressal of grievances.
∙ Advise the Government on all policy matters affecting women.
∙ Reference: UPSC 2015 question number
Q.7) Consider the following statements regarding the Central Bureau of Investigation:

1. CBI can take suo motu cognizance in a matter of corruption involving government officials
and incidents of violent crime.
2. The Central Bureau of Investigation functions under the Department of Personnel, Ministry
of Personnel, Pension and Public Grievances.

Which of the statements given above is/are correct?

a) 1 only
b) 2 only
c) Both 1 and 2
d) Neither 1 nor 2
Ans: B

∙ Statement 1 is incorrect: Unlike the National Investigation Agency(NIA), CBI cannot take suo motu cognizance of a case
in a state — whether in a matter of corruption involving government officials of the Centre and PSU staff or an incident of
violent crime. In order to take up corruption cases involving central government staff, it either needs general consent of
the state government or specific consent on a case-to-case basis. For all other cases, whether involving corruption in the
state government or an incident of crime, the state has to request an investigation by the CBI, and the Centre has to
agree to the same. In case the state does not make such a request, the CBI can take over a case based on the orders of
the High Court Concerned or the Supreme Court.
∙ Statement 2 is correct: Central Bureau of Investigation (CBI) is the premier investigating police agency in India. It
functions under the superintendence of the Deptt. of Personnel, Ministry of Personnel, Pension & Public Grievances,
Government of India - which falls under the prime minister’s office. It is also the nodal police agency in India which
coordinates investigations on behalf of Interpol Member countries.
Q.8) Recently “National eVidhan Application (NeVA)”, a Mission
Mode Project for Digital Legislatures was in the news is developed
PYQ ASKED ON SIMILAR LINE
by:

a) Ministry of Legal affairs


b) Ministry of Statistics and Program Implementation
c) Ministry of Parliamentary Affairs
d) Ministry of Home affairs
Ans: C

∙ The Ministry of Parliamentary Affairs (MoPA) has developed a “National eVidhan Application (NeVA)”, a Mission Mode
Project for Digital Legislatures to make the functioning of all Legislative Houses in the country paperless. National
eVidhan Application(NeVA) is a Mission Mode Project (MMP) included in the Digital India Programme. To make the
functioning of all Legislative Houses in the country paperless on the theme of ‘One Nation –One Application’.
∙ To transform all State Legislatures into ‘DIGITAL HOUSES’ so as to enable them to transact entire Government Business on
digital platforms including information exchange with the State Government Departments in digital mode. The
application has also enabled provisions for onboarding the two Houses of Parliament.
∙ The funding for e-Vidhan is provided by the MoPA. It is on the pattern of Central Sponsored Scheme, i.e. 60:40 for States;
90:10 for North East & hilly States and 100% for UTs. In 2021, Bihar Legislative Council became the first House in the
country to transit to NeVA platform completely and conducted the Winter Session, 2021 on the NeVA platform in
paperless mode.
Q.9) Consider the following statements regarding office of PYQ ASKED ON SIMILAR LINE
Governor:
Q. Consider the following statements: [2018]
1. Governor is the constitutional head of the state, bound by the No criminal proceedings shall be instituted
advice of the council of ministers of that state. against the Governor of a State in any court
2. The Governor has Constitutional discretion to invite the leader during his term of office.
of the largest party to form the government when no party is in The emoluments and allowances of the
majority.
Governor of a State shall not be diminished
3. In the Nabam Rebia judgement (2016) Supreme Court held that
the exercise of Governor’s discretion Article 163 is limited and
during his term of office.
his choice of action should not be arbitrary or fanciful. Which of the statements given above is/are
correct?
Which of the statements given above is/are correct? (a) 1 only
(b) 2 only
a) 1 and 2 only (c) Both 1 and 2
b) 1, 2 and 3 (d) Neither 1 nor 2
c) 2 and 3 only
d) 1 and 3 only
Ans: D

∙ Statement 1 is correct: The Governor’s appointment, his powers and everything related to the office of Governor have
been discussed under Article 153 to Article 162 of the Indian Constitution. Governor is the constitutional head of the
state, bound by the advice of his council of ministers. He functions as a vital link between the Union Government and the
State Government.
∙ Statement 2 is incorrect: The governor has two categories of discretion, Constitutional Discretion and discretion
mentioned in the Constitution. Situational Discretion is a hidden discretion derived from the exigencies of a prevailing
political situation. Appointment of a chief minister when no party has a clear majority in the state legislative assembly or
when the current chief minister dies unexpectedly and there is no obvious successor, is a situational discretion. Other
situational discretion includes Dismissal of the council of ministers when it is unable to demonstrate the confidence of
the state legislative assembly. Dismissal of state legislative assembly when the council of ministers lose their
majority.According to the Constitution the governor has constitutional discretion in the followingcases:Reservation of a
bill passed by legislative assembly for the consideration of the President.Recommendation for the imposition of the
President’s Rule in the concerned state.When he is exercising his functions as the administrator of an adjoining union
territory.Determining the amount payable by the Government of Assam, Meghalaya, Tripura and Mizoram to an
autonomous Tribal District Council as royalty accruing from licences for mineral exploration.Seeking information from the
chief minister with regards to the administrative matters of the state.
∙ Statement 3 is correct: The Constitution Bench judgement of the Supreme Court in Nabam Rebia versus Deputy Speaker
on July 13, 2016, held that a Governor is bound to convene a meeting of the Assembly for a floor test on the
recommendation of the Cabinet. A Governor cannot employ his “discretion”, and should strictly abide by the “aid and
advice” of the Cabinet to summon the Assembly for a floor test. Governor’s discretionary powers are limited to specified
areas like giving assent or withholding/referring a Bill to the President or appointment of a Chief Minister or dismissal of a
government which has lost of confidence but refuses to quit, etc.
Q.10) Recently a demand for creating Indian Legal Services is in the news. Which of the
following are the concerns with the current practice of appointing outsiders as
Secretary-Generals?

1. Against the principle of separation of powers.


2. Lack of knowledge of the functioning of the legislature.
3. Impact Legislatures' role of holding the executive accountable.

Select the correct answer using codes given below:

a) 1 and 2 only
b) 1, 2 and 3
c) 2 and 3 only
d) 1 and 3 only
Ans: B

∙ Concerns with the current practice of appointing outsiders as Secretary-Generals:


∙ Against the principle of separation of powers: Appointing serving or retired civil servants to the post of
Secretary-General amounts to the breach of the principle of separation of power as envisaged under the Indian
Constitution and also upheld by the judiciary as a basic structure of the Constitution. Article 98 of the Constitution by
providing for two separate secretariats emphasizes the principle that the secretariats should be independent of the
executive government. A separate and independent secretariat marks a feature of a functioning parliamentary
democracy. Appointing civil servants to the post of Secretary-General, could undermine the independence of the
Secretariat.
∙ Lack of knowledge of the functioning of the legislature: The Secretary-General is entrusted with a wide range of duties
and functions which necessitate vast knowledge and rich experience of Parliamentary procedures, practices and
precedents. Most of the civil servants may lack this expertise.
∙ Impact Legislatures' role of holding the executive accountable: In a parliamentary polity like India, apart from passing
legislations, the Parliament is also expected to scrutinise the executive’s administrative behaviour. A strong Parliament
means a more accountable executive. Appointing former bureaucrats to head important legislature bodies like the
Secretariat could lead to possible conflict of interests.
Q.11) Consider the following statements regarding Electronically Transmitted Postal Ballot System (ETPBS):

1. In ETPBS postal ballots are sent by post to registered service voters for voting.
2. An NRI can vote in the constituency in which his/her place of residence is located.

Which of the statements given above is/are correct?

a) 1 only
b) 2 only
c) Both 1 and 2
d) Neither 1 nor 2
Ans: C

∙ Statement 1 is correct: The Conduct of Election Rules, 1961 was amended in 2016 to allow service voters to use the
ETPBS. Under this system, postal ballots are sent electronically to registered service voters. The service voter can then
download the ETPB (along with a declaration form and covers), register their mandate on the ballot and send it to the
returning officer of the constituency via ordinary mail. The post will include an attested declaration form (after being
signed by the voter in the presence of an appointed senior officer who will attest it).
∙ Statement 2 is correct: through the Representation of the People (Amendment) Act, 2010, eligible NRIs who had stayed
abroad beyond six months were allowed to vote, but only in person at the polling station where they have been enrolled
as an overseas elector. An NRI can vote in the constituency in his/her place of residence, as mentioned in the passport, is
located. He/She can only vote in person and will have to produce her passport in original at the polling station for
establishing identity.
Q.12) Recently, the Montreux Convention was seen in the news PYQ ASKED ON SIMILAR LINE
related to:
Q. What is/are the importance/ importances of the ‘United
a) Reduce the Methane pollution. Convention to Combat Desertification’? [2016]
1.It aims to promote effective action through innovative
b) Control the Transboundary Movements of Hazardous national programmes and supportive international
Wastes. partnerships
c) legal framework for all marine and maritime activities. 2.It has a special/particular focus on South Asia and North
Africa regions, and its Secretariat facilitates the allocation of
d) The Regime of the Strait. major portion of financial resources to these regions.
3.It is committed to bottom-up approach, encouraging the
participation of local people in combating the
desertification.
Select the correct answer using the code given below.
a) 1 only
b) 2 and 3 only
c) 1 and 3 only
d) 1, 2 and 3
Ans: D

∙ Option D is correct: Black sea strait also known as the Turkish Straits, the Black Sea Straits are the Bosporus and
Dardanelles strait and connect the Aegean Sea and the Black Sea via the Sea of Marmara. It is the only passage through
which the Black Sea ports can access the Mediterranean and beyond.
∙ Montreux Convention is the international agreement was signed by Australia, Bulgaria, France, Greece, Japan, Romania,
Yugoslavia, the United Kingdom, the Soviet Union and Turkey and has been in effect since November 1936 is Regarding
the Regime of the Straits is an international agreement governing the Bosporus and Dardanelles Straits in Turkey.
∙ This pact gives authority to Turkey to have control over both the Black Sea Straits. In the event of a war, the pact gives
Ankara the right to regulate the transit of naval warships and to block the straits to warships belonging to the countries
involved in the conflict. Article 19 of the treaty contains an exception for the countries on the Black Sea that can
effectively undermine Turkey’s power in blocking the Russian warships entering or exiting the Black Sea.
∙ The Article 19 says, “Vessels of war belonging to belligerent powers, whether they are Black Sea Powers or not, which
have become separated from their bases, may return thereto.” That means warships can return to their actual bases
through the passage and Turkey cannot prevent it. The term applies to Russian fleets also currently in the Black Sea that
belong to a base in the Mediterranean or Baltic Sea. Russia is free to take them out of the Black Sea.
Q.13) Recently, PM-DAKSH (Pradhan Mantri Dakshta Aur Kushalta
Sampann Hitgrahi) was in the news, with the aim of: PYQ ASKED ON SIMILAR LINE

a) Digital learning portal for Persons with disabilities


b) To Procure forest minor produce
c) Skill development training programme to targeted groups.
d) To Skill Self defence training for Women.
Ans: C

∙ PM DAKSH (Pradhan Mantri Dakshta Aur Kushalta Sampann Hitgrahi) is being implemented from the year 2020-21.
Under this, eligible target groups are provided with the skill development training programmes on Short Term Training
Program; and Up-Skilling/Reskilling; Entrepreneurship Development Programme, and Long Term Training Programme.
∙ These training programmes are being implemented through the government training institutions, sector skill councils
which have been constituted by the Ministry of Skill Development and Entrepreneurship, and other credible institutions.
Marginalised persons of SC, OBC , EBC, Denotified tribes, Sanitation workers including waste pickers, manual scavengers,
transgenders and other similar categories will be Eligible for this scheme.
∙ The program is implemented by the 3 Corporations under the Ministries as:
∙ National Scheduled Castes Finance and Development Corporation (NSFDC),
∙ National Backward Classes Finance and Development Corporation (NBCFDC),
∙ National Safai Karamcharis Finance and Development Corporation (NSKFDC).
Q.14 Which of the following countries share borders PYQ ASKED ON SIMILAR LINE
with the Arctic Ocean?

1. Russia
2. Norway
3. Iceland
4. Canada
5. United States

Select the correct answer using the code given below:

a) 1 , 2 and 3 only
b) 2 ,4 and 5 only
c) 1 , 3 and 5 only
d) All of the above
ANS: D
Why in News?

The Ministry of Earth Science (MoES) has released India's Arctic policy titled 'India and the Arctic: building a partnership
for sustainable development’.

India’s Arctic policy approach:


∙ Deepen cooperation between India and countries of Arctic region under various Arctic forums and Increase India’s
participation in the Arctic Council.Harmonize polar research with the third pole- the Himalayas.Contribute better analysis,
prediction, and coordinated policymaking on the implications of ice melting in the Arctic on India's economic, military,
and strategic interests.
∙ The policy will involve multiple stakeholders, including academia, research community, business, and industry. National
Center for Polar and Ocean Research (NCPOR) in Goa, an autonomous institute under the Ministry of Earth Sciences,
is the nodal institution for India’s Polar research programme, which includes Arctic studies.Need of Arctic policy is given
below-
∙ Rising Sea Level: The land ice loss due to global warming is a major contributor to global sea-level rise, it can have a
significant impact on marine biodiversity. From 1971 till 2019, Arctic snow cover and extent of Arctic-sea ice have shrunk
by 21 percent and 43 percent respectively.
∙ Monsoons: Changes in the Arctic have a huge impact on global weather, climate and ecosystems including Indian
monsoons which impacts agriculture, primary source of livelihood for about 58 percent of population and contributes
around 20 percent to the GDP.
∙ Glacial melt in Himalayas: It’ll help the scientific community to better understand the glacial melt in Himalayas, which is
the source of Ganga and Brahmaputra rivers, basins of which support a population of about 600 million. Etc.
Q.15) Consider the following statements regarding ‘Scheduled PYQ ASKED ON SIMILAR LINE
Tribe’:
Q. If a particular area is brought under the Fifth
1. The Constitution defines the criteria for recognition of Schedule of the Constitution of India, which one
Scheduled Tribes under article 366. of the following statements best reflects the
2. Fifth schedule deals with the administration of the tribal areas consequence of it? [2022]
in Assam, Meghalaya, Tripura and Mizoram.
3. Darlong community belongs to tribal communities of Arunachal (a)This would prevent the transfer of land of
Pradesh. tribal people to non-tribal people.

(b)This would create a local self-governing body


Which of the statements given above is/are not correct?
in that area.
a) 1 and 3 only (c)This would convert that area into a Union
b) 1, 2 and 3 Territory.
c) 1 and 2 only
d) 2 and 3 only (d)The State having such areas would be
declared a Special Category State.
Ans: B

∙ Statement 1 is incorrect: The Constitution does not define the criteria for recognition of Scheduled Tribes (ST) and hence
the definition contained in 1931 Census was used in the initial years after independence.However, Article 366(25) of the
Constitution only provides a process to define Scheduled Tribes (ST): “Scheduled Tribes means such tribes or tribal
communities or parts of or groups within such tribes or tribal communities as are deemed under Article 342 to be
Scheduled Tribes for the purposes of this Constitution.
∙ Statement 2 is incorrect: The 5th Schedule lays out provision for Administration and Control of Scheduled Areas and
Scheduled Tribes in states other than Assam, Meghalaya, Tripura and Mizoram (AMTM). The 6th Schedule deals with the
administration of the tribal areas in Assam, Meghalaya, Tripura and Mizoram. There are over 705 tribes which have been
notified. The largest number of tribal communities are found in Odisha.
∙ Statement 3 is incorrect: Darlong is a tribal community of Tripura, which has a population of 11,000. The community has
a high prevalence of education and cultural activity and its members of the community serve in senior positions in the
local administration. For example, a tribal musicologist and Rosem (a tribal instrument) maestro Thanga Darlong was
awarded the prestigious Padma Shri a few years ago for his contributions to culture.
Q.16 Consider the following statements with reference PYQ ASKED ON SIMILAR LINE
to the Savitribhai Phule:

1. She was the India’s first generation modern feminists


for her significant contributions in ensuring equal
education opportunities under the British raj.
2. She became the first female teacher in India in 1848
and opened a school for girls.
3. She wrote a poem entitled “Go, Get Education”

Which of the statements given above is/are correct?

a) 1 and 2 only
b) 2 and 3 only
c) 1 and 3 only
1, 2 and 3
ANS: D

∙ Statement 1 is correct: Savitribai Phule was an Indian social reformer, educationalist and poet from Maharashtra. She was
the India’s first generation modern feminists for her significant contributions in ensuring equal education opportunities
under the British raj. She established the Mahila Seva Mandal to raise awareness for issues concerning women’s rights.
∙ Statement 2 is correct: Savitribai Phule became the first female teacher in India in 1848 and opened a school for girls
along with her husband, social reformer Jyotirao Phule. She opened a care centre called Balhatya Pratibandhak Griha for
pregnant rape victims and helped deliver and save their children. Her friend Fatima Begum Sheik was the first Muslim
woman teacher of India.
∙ Statement 3 is correct: Savitribai Phule was also an author and poet. She published Kavya Phule in 1854 and Bavan Kashi
Subodh Ratnakar in 1892, and also a poem entitled “Go, Get Education” in which she encouraged those who are
oppressed to free themselves by obtaining an education.
Q.17) “World Energy Transitions Outlook” PYQ ASKED ON SIMILAR LINE
is published by

a) International Renewable Energy Agency


(IRENA)
b) International Energy Agency (IEA)
c) World Energy Council
d) Alliance to save energy
ANS: A

∙ International Renewable Energy Agency (IRENA) launched the World Energy Transitions Outlook
2022
∙ What is the International Renewable Energy Agency (IRENA)? It is an intergovernmental organisation, it was
officially founded in Bonn, Germany, in January 2009. It has 167 members and India is the 77th Founding Member of
IRENA. It has its headquarters in Abu Dhabi, United Arab Emirates
∙ The International Energy AgencyIt is a Paris-based autonomous intergovernmental organisation established in the
framework of the Organisation for Economic Co-operation and Development (OECD) in 1974 in the wake of the 1973 oil
crisisIt is best known for the publication of its annual World Energy OutlookIn March 2017, after a series of intensive
consultations with all the relevant ministries, India joined the IEA as an association country
∙ The World Energy Council It is a global forum for thought-leadership and tangible engagement with headquarters in
London. Its mission is 'To promote the sustainable supply and use of energy for the greatest benefit of all people'.The
World Energy Council hosts the World Energy Congress, which is the world's largest and most influential energy event
covering all aspects of the energy agenda.The World Energy Council's publications include annual releases like the World
Energy Trilemma Index,India is a member.
∙ The Alliance to Save Energy is a bipartisan, non-profit coalition of business, government, environmental, and consumer
groups based in Washington, D.C.
Q.18) Consider the following statements regarding ‘New Census rules’:

1. New Census rule allows online self-enumeration in the upcoming Census and National Population Register (NPR).
2. Print media, radio, audio-visuals and posters have been added to the list of modes for ensuring wide publicity for the
exercise.

Which of the statements given above is/are correct?

a) 1 only
b) 2 only
c) Both 1 and 2
d) Neither 1 nor 2
Ans: A

∙ Statement 1 is correct: The government of India has amended Census Rules, 1990, to include “electronic form” and
“self-enumeration” in the schedule of questions to be asked during census enumeration. It will allow online
self-enumeration in the upcoming Census and National Population Register (NPR). NPR is a Register of usual residents of
the country and the objective is to make a comprehensive identity database of every usual resident in the country.
∙ Statement 2 is incorrect: Census Act, 1948 has Section 8 given powers to the enumerator to ask questions regarding the
Census and makes it mandatory for respondents to answer, with some exceptions. Also, “print media, electronic media,
social media”, have been added to the list of modes for ensuring wide publicity for the exercise. Before the new census
rules, lists only included radio, audio-visuals and posters. Earlier, the 1st phase of Census 2021, an NPR exercise
scheduled to be held in 2020 was postponed indefinitely due to Covid-19 crisis. Population Census is the process of
collecting, compiling, analysing, disseminating demographic, economic and social data pertaining, at a specific time, of all
persons in a country or a part of a country.
Extra edge by Onlyias
Census history:
A systematic and modern population census, in its present
form, was conducted non synchronously between 1865 and
1872 in different parts of the country.
This effort culminating in 1872 has been popularly labelled as
the first population census of India.
However, the first synchronous census in India was held in
1881. Since then, censuses have been undertaken
uninterruptedly once every ten years.
Q.19) Consider the following statements regarding Criminal Procedure (Identification) Act, 2022:

1. The Act requires the details collected to be retained in digital form for not more than 25 years of the date of
collection.
2. As per the Act, resistance or refusal to give details will be considered an offence under the Indian Penal
Code, 1860.
3. The Act expanded rule-making power to the central government.

Which of the statements given above is/are correct?

a) 1 and 2 only
b) 2 and 3 only
c) 1, 2 and 3
d) 1 and 3 only
Ans: B

∙ Statement 1 is incorrect: The Criminal Procedure (Identification) Act, expands the type of data that may be collected,
persons from whom such data may be collected, and the authority that may authorise such collection. The Act requires
the details collected to be retained in digital or electronic form for 75 years from the date of collection. The record may
be destroyed in case of persons who have not been previously convicted, and who are released without trial, discharged,
or acquitted by the court, after exhausting all legal remedies.
∙ Statement 2 is correct: As per the Act, resistance or refusal to give details will be considered an offence under the Indian
Penal Code, 1860. In case of such resistance or refusal, police officers or prison officers may collect details in the manner
prescribed under rules made by the state government or the central government. Under the Act, a Magistrate may direct
a person to give details for the purpose of an investigation or proceeding under the CrPC (Code of Criminal Procedure).
∙ Statement 3 is correct: The Criminal Procedure (Identification), Act seeks to replace the Identification of Prisoners Act,
1920. The Act vested rule-making power only in the state government. The Amendment Act extends this power to the
central government as well. The central or state government may make rules on various matters like the manner of
collecting details.
Q.20) Which of the following are the causes of the Paika Rebellion PYQ ASKED ON SIMILAR LINE
(1817-1825)?

1. Paikas lost their land because of the Land revenue system of Q. With reference to the history of India,
British colonial rule. “Ulgulan” or the Great Trumult is the description
2. Paikas were facing a problem of handling the new currency of which of the following events?
system which converted currency as cowrie to rupee. (a) The Revolt of 1857
3. British salt policy leads to bringing them in disadvantaged from (b) The Mappila Rebellion of 1921
their traditional right of making Salt. (c) The Indigo REvolt of 1859-60
(d) Birsa Munda’s Revolt of 1899-1900
Select the correct answer using codes given below:

a) 1, 2 and 3
b) 2 and 3 only
c) 1 and 2 only
d) 1 and 3 only
Ans: A

∙ Paika rebellion was an armed rebellion that took place in Odisha against the British East India Company (EIC). It took
place nearly 40 years before the first sepoy mutiny. The Paikas were peasant militias of the Gajapati rulers of Odisha who
offered military services to the king. They owned rent-free land that had been given to them for their military service to
the Kingdom of Khurda.
∙ The British established themselves in Odisha when the EIC dethroned the King of Khurda, Raja Mukunda Deva in 1803.
The Paikas under Bakshi Jagabandhu Bidyadhar, the hereditary chief of the militia army of the Gajapati King, rose in
rebellion, taking support of tribals and other sections of society.
∙ Causes of Paika Rebellion: Land revenue system: The advent of the British and establishment of colonial rule brought
new land revenue settlements, which led to the Paikas losing their estates.
∙ British exploitative policies: With the removal of the King of Khurda, exploitative policies of the British administration
became increasingly intolerable for the people of Odisha. The continuous interference in the economy and revenue
systems led to exploitation and oppression of the peasants and farmers eventually triggering a rebellion against the
British.
∙ New currency system: The British changed the currency system from cowrie to rupee. The villagers faced a lot of
problems in handling the new currency and they were grossly exploited by the local mahajanas.
∙ British salt policy: The long sea coast of Odisha produced a huge quantity of salt which was used by the people of this
land freely. However, the British authorities deprived the zamindars and the local people of the coastal region of their
traditional rights to manufacture salt.
Q.21) Recently, Sohrai painting was in the news, PYQ ASKED ON SIMILAR LINE
refers to: Kalamkari painting refers to
(a)a hand-painted cotton textile in South
a) The hand painted textile in South India India
(b)a handmade drawing on bamboo
b) Miniature art prevalent in Rajasthan handicrafts in North-East India
c) Indigenous art form of painting from (c)a block-painted woollen cloth in Western
Jharkhand Himalayan region of India
(d)a hand-painted decorative silk cloth in
d) The decorative painted silk in West Bengal North-Western India
Ans: C

∙ Sohrai painting is an indigenous art form of painting used by tribal women in the district of Hazaribagh, Jharkhand. The
paintings are done on mud walls to celebrate the cattle and to welcome the harvest. The women clean their houses and
use murals of Sohrai art to decorate their walls.
∙ This art form has continued since 10,000-4,000 BC. Earlier, It was prevalent mostly in caves, but later shifted to houses
with mud walls.
∙ Key Features of Sohrai Art are as following:
∙ The Sohrai art form can be both monochromatic or colourful. The people coat the wall with a layer of white mud, and
while the layer is still wet, they draw with their fingertips on it.
∙ Their designs range from flowers and fruits to various other nature-inspired designs. The cow dung that was earlier used
to cake the walls of the house is used to add colour.
∙ The artists are spontaneous in their drawings. The designs of art are generally drawn from the artist's memory. The
personal experience of the artist and their interaction with nature are the biggest influence.
∙ Sohrai Khovar painting received the Geographical Indication(GI) tag in 2020.
Q.22) Consider the following statements regarding Sahitya Akademi:

1. Sahityotsav is an inclusive literature festival organised by Sahitya Akademi.


2. Sahitya Akademi award gives 22 awards annually to literary works in the 22 official languages.
3. Sahitya Akademi as a government institution works under the ministry of Culture.

Which of the statements given above is/are correct?

a) 1 only
b) 2 and 3 only
c) 1 and 2 only
d) 3 only
Ans: A

∙ Statement 1 is correct: Sahithyolsav is a Festival of Letters of Sahitya Akademi. It is India’s most inclusive literature
festival. The sahityotsav festival will be a part of the celebrations to commemorate the 75th anniversary of India’s
Independence. During the festival, the prestigious Sahitya Akademi Awards will also be presented.
∙ Statement 2 is incorrect: Sahitya Akademi award was established in 1954. It is an honour of literature that is annually
conferred by Sahitya Akademi, India’s National Academy of letters. The Akademi gives 24 awards annually to literary
works in the languages it has recognized, and an equal number of awards to literary translations from and into the
languages of India. Besides the 22 languages enumerated in the Constitution of India, the Sahitya Akademi has
recognised English and Rajasthani as languages in which its programme may be implemented. The Sahitya Akademi
award is the second-highest literary honour by the Government of India, after the Jnanpith award.
∙ Statement 3 is incorrect: in 1954, Ministry of Culture established Sahitya Akademi, India’s National Academy of Letters.
Though it was set up by the Government, but functions as an autonomous organisation. In 1960 it was registered as a
society under the Societies Registration Act, 1860. It is an organisation dedicated to the promotion of literature in the
languages of India and located in Rabindra Bhawan near Mandi House in Delhi.
Q.23) Which of the following organization publishes “State of India’s Environment
Report 2022”?

a) The Energy and Resources Institute (TERI)


b) Centre for Environmental Planning and Technology (CEPT)
c) Centre for Environment Education (CEE)
d) Centre for Science and Environment (CSE)
ANS: D

∙ Centre for Science and Environment is a public interest research and advocacy organisation based in New Delhi.
∙ Centre for Science and Environment (CSE) released the State of India’s Environment Report 2022.
∙ The report is the annual publication of the Centre for Science and Environment, and Down To Earth (magazine).
∙ The report focuses on climate change, migration, health and food systems. It also covers biodiversity, forest and wildlife,
energy, industry, habitat, pollution, waste, agriculture and rural development.
Q.24) Consider the following pairs regarding India's Musical PYQ ASKED ON SIMILAR LINE
Instruments:
With reference to Dhrupad, one of the major
Instrument Description traditions of India that has been kept alive for
1. Ghan Vadya Self-vibrators, owing to their elastic centuries, which of the following statements
nature are correct?
2. Tat Vadya Sound is produced by striking the 1.Dhrupad originated and developed in the
animal skin Rajput kingdoms during the Mughal period.
3. Avanaddha Vadya Sound is produced by the vibration of 2.Dhrupad is primarily a devotional and
a string or chord spiritual music.
3.Dhrupad Alap uses Sanskrit syllables from
Which of the pairs given above is/are correct? Mantras.
Select the correct answer using the codes given
a) 1 only below:
b) 2 and 3 only (a) 1 and 2 only
c) 2 only (b) 2 and 3 only
d) 1 and 3 only (c) 1, 2 and 3
(d) None of the above is correct
Ans: A

∙ Pair 1 is correct: Ghan Vadya (idiophones or solid instruments) Ghan Vadya are self-vibrators, i.e. owing to their elastic
nature, they have a sonority of their own, which is emitted in waves when they are struck, plucked, or stimulated by
friction or air. These instruments are not capable of producing definite pitches that are required for creating a melody.
That is why their use is limited in classical music. In the Sun temple of Konarak, Orissa, we see this large sculpture nearly
8 f1. high of a lady playing the Jhanj. Example: Bell, Bhajan Chakkalu, Bhuanj, etc.
∙ Pair 2 is incorrect: In Tat Vadya (chordophones or stringed instruments), Sound is produced by the vibration of a string or
chord. These vibrations are caused by plucking or by bowing (Ravanastram is one of the earliest known bowed
instruments)on the string which has been pulled taut. Representation of these can be found in many sculptures and
murals of olden days, as for example, in the Bharhut and Sanchi Stupa, the reliefs of Amaravati and so on. Mentions of
Yazh are found in old Tamil texts from the 2nd century A.D. Example: Santoor, sarangi, dilruba, esraj, Kamaicha etc.
Pandit Shiv Kumar Sharma is the famous Santoor player, and Ustad Sultan Khan is a renowned player of the sarangi.
∙ Pair 3 is incorrect: In Avanaddha Vadya (membranophones or percussion instruments) Sound is produced by striking the
animal skin which has been stretched across an earthen or metal pot or a wooden barrel or frame. The earliest
references to such instruments have been found in the Vedas where there is mention of Bhumi Dundhubhi. Types: They
have been divided into different categories on the basis of their shapes Oordhwaka: Like Tabla pair and Chenda.
Prominent musicians playing the tabla today are-Ustad Alia Rakha Khan, Zakir Hussain, Shafat Ahmed and Samata Prasad.
∙ Ankya: Today, in this variety, the Mridangam, Pakhawaj, Khol, etc. are prominent.
∙ Alingya: The Duff, Dufflies, etc.Damaru types: Huddaka of Himachal Pradesh, Timila of the southern region.
Q.25) About IPCC (Intergovernmental Panel on Climate PYQ ASKED ON SIMILAR LINE
Change) which of the following are statements correct?
Q. What is ‘Greenhouse Gas Protocol’? [2016]
1. It was set up by the World Meteorological Organisation a) It is an international accounting tool for
(WMO) and United Nations Environment Programme government and business leaders to understand,
(UNEP). quantify and manage greenhouse gas emissions.
2. IPCC assessments provide a scientific basis for b)It is an initiative of the United Nations to offer
financial incentives to developing countries to
governments at all levels to develop climate related
reduce greenhouse gas emissions and to adopt
policies
eco-friendly technologies
3. IPCC recently was a part of implementation of the c)It is an intergovernmental agreement ratified by
National Mission for Clean Ganga. all the member countries of the United Nations to
reduce greenhouse gas emissions to specified levels
Choose the correct code among the following: by the year 2022
d)It is one of the multilateral REDD+ initiatives
a) 1 and 2 only hosted by the World Bank
b) 2 and 3 only
c) 1 and 3 only
d) 1, 2 and 3
ANS: A

∙ Statement 1 is correct: Intergovernmental Panel on Climate Change is the international body for assessing the science
related to climate change.It was set up in 1988 by the World Meteorological Organisation (WMO) and United Nations
Environment Programme (UNEP) to provide policymakers with regular assessments of the scientific basis of climate
change, its impacts and future risks, and options for adaptation and mitigation.
∙ Statement 2 is correct: IPCC assessments provide a scientific basis for governments at all levels to develop climate
related policies, and they underlie negotiations at the UN Climate Conference – the United Nations Framework
Convention on Climate Change (UNFCCC).
∙ Statement 3 is incorrect: IPCC has not been part of implementation of NMCG (National Mission for Clean Ganga), as IPCC
mostly confine itself to publishing data and is not involved in project implementations.
Q.26 With reference to Santali language, Consider the PYQ ASKED ON SIMILAR LINE
following statements:

1. In 1925 Pandit Raghunath Murmu invented the script ‘Ol


Chiki’, to write Santali.
2. Santali language has been added to the eighth schedule of
the constitution by the 71st Amendment Act of 1992.

Which of the statements given above is/are correct?

a) 1 only
b) 2 only
c) Both 1 and 2
d) Neither 1 nor 2
Ans: A

∙ Statement 1 is correct: Ol Chiki script–the word ol means “write,” and chiki means “script”–was invented by Pandit
Raghunath Murmu in 1925 to write Santali. Ol Chiki is alphabetic, and does not share any of the syllabic properties of the
other Indic scripts. The script contains 30 letters and five basic diacritics (marks placed above or below (or sometimes
next to) a letter in a word to indicate a particular pronunciation).
∙ Statement 2 is incorrect: By the 92nd Constitutional Amendment Act, 2003 added Santhali to the eighth schedule of the
Constitution of India, which lists the official languages of India, along with the Bodo, Dogri and Maithili languages. This
addition meant that the Indian government was obligated to undertake the development of the Santali language.
Through the Eighth Schedule, the government allows students appearing for school-level examinations and entrance
examinations for public service jobs to use the language.
Q.27 Consider the following statements:
PYQ ASKED ON SIMILAR LINE
1. BIMSTEC is a regional multilateral organization that
connects South and Southeast Asia.
2. Bangladesh, Bhutan, India, Nepal, Sri Lanka, Myanmar
and Thailand are members of BIMSTEC.
3. BIMSTEC is home to around 1.5 billion people that share
around 22% of the global population.

Which of the statements given above is/are correct?

a) 2 only
b) 1 and 2 only
c) 2 and 3 only
d) 1, 2 and 3 only
Ans: D
Q.28) Consider the following statements regarding to PYQ ASKED ON SIMILAR LINE
Motor Vehicles Agreement:
With reference to the "G20 Common
1. Recently India, Nepal and Bangladesh finalised the Framework", consider the following statements:
implementation of the Motor Vehicles Agreement. 1.It is an initiative endorsed by the G20 together
2. Its purpose is to provide seamless people-to-people with the Paris Club.
contact and enhance economic interaction. 2.Itis an initiavtive to support Low Income
3. The ASEAN Development Bank has been providing Countries with unsustainable debt.
technical, advisory, and financial support to the Which of the statements given above is/are
Bangladesh-Bhutan-India-Nepal Motor Vehicles correct?
Agreement initiative. (a) 1 only
(b) 2 only
Select the correct answer using the code given below: (c) Both 1 and 2
(d) Neither 1 nor 2
a) 1, 2 and 3 only
b) 3 and 4 only
c) 1 and 2 only
d) 2, 3 and 4 only
Ans: C

∙ Statement 1 is correct: Recently, India, Nepal and Bangladesh have finalized a Memorandum of Understanding for
implementing the Bangladesh-Bhutan-India-Nepal (BBIN) Motor Vehicles Agreement. Bangladesh, Bhutan, India and
Nepal(BBIN) had signed the Motor Vehicles Agreement(MVA) in 2015 at Thimphu, Bhutan.
∙ Statement 2 is correct: Its purpose is not only to provide seamless people-to-people contact but also to enhance
economic interaction by facilitating cross border movement of people and goods. As per the agreement, member
countries would allow vehicles registered in the other countries to come to their territory under some terms and
conditions and Customs and tariffs would be decided by the respective countries and these would be finalized at bilateral
and trilateral forums.
∙ Statement 3 is incorrect: The Asian Development Bank (ADB) has been providing technical, advisory, and financial
support to the BBIN MVA initiative as part of its assistance to the South Asia Subregional Economic Cooperation (SASEC)
program. Tiven Bhutan’s “current infrastructure” and first priority to remaining a “carbon- negative” country, it said it
would not be possible to consider joining the MVA. Operationalising the MVA by concluding the Passengers. The Cargo
Protocol will help realize the full capacity of trade and people to people connectivity between the BBIN countries by
fostering greater sub- regional cooperation.
Q.29) The PACER Initiative was recently seen in news in the context of:

a) Increasing the pace of installation of rooftop solar photovoltaic cell


b) Cryosphere research
c) Curbing plastic pollution in the urban water bodies
d) Increasing rate of plantation for land reclamation
ANS: B

∙ About PACER Initiative:


∙ The Polar Science and Cryosphere Research (PACER) Initiative was launched by the Ministry of Earth Science.
∙ The objective is to study various aspects relating to Polar and the Cryosphere with special emphasis on
the Antarctic, Arctic and Glaciers of the Himalayas.
∙ The important programmes under the initiative include: the Antarctic program, Indian
∙ Arctic program, Southern Ocean program and Cryosphere and Climate
∙ program. National Center for Polar and Ocean Research (NCPOR) is the implementing agency.
∙ he initiative has been approved for continuation during 2021- 2026.
Q.30) With reference to the International Fund for Agricultural PYQ ASKED ON SIMILAR LINE
Development, consider the following statements:
With reference to the "United Nations
1. It is a specialised United Nations agency. Credentials Committee", consider the following
2. It helps people to expand their businesses. statements:
3. It publishes a Rural Development Report every year. 1.It is a committee set up by the UN Security
Council and works under its supervision.
Which of the statements given above is/are correct? 2.It traditionally meets in March, June and
September every year.
a) 1 and 3 only 3.It assesses the credentials of all UN members
b) 1 and 2 only before submitting a report to the General
c) 2 and 3 only Assembly for approval.
d) 1, 2 and 3 Which of the statements given above is/are
correct?
(a) 3 only
(b) 1 and 3
(c) 2 and 3
(d) 1 and 2
Ans: D

∙ All the statements are correct: International Fund for Agricultural Development is a specialized United Nations agency
created in 1977 engaged in providing grants and loans with low interest for allied projects. The IFAD works with rural
people allowing them to enhance their food security, improve nutrition and raise their incomes.
∙ The International Fund for Agricultural Development also helps people to expand their businesses. The organization is an
outcome of the World Food Conference of 1974 and headquartered in Rome and headed by a President.The objectives of
the IFAD are three-fold To increase the productive capacity of poor people.To increase benefits for them from market
participation.To strengthen the environmental sustainability & climate resilience of their economic activities.
∙ The International Fund for Agricultural Development has 177 member countries. IFAD brings out the Rural Development
Report every year. IFAD’s Rural Development Report 2021 is focused on rural livelihoods in the context of food systems
transformation. The report promotes equitable livelihoods for rural people, who are front and centre in transforming
food systems, alongside the need to improve nutrition and protect the environment.
Q.31) The term Most Favoured Nation is associated with which PYQ ASKED ON SIMILAR LINE
of the following organizations?
Which one of the following statements best
a) World Health Organization
describes the 'Polar Code'?
b) World trade organization
(a)It is the international code of safety for ships
c) World economic forum
operating in polar waters.
d) World Bank
(b)It is the agreement of the countries around the
North Pole regarding the demarcation of their
territories in the polar region.
(c)It is a set of norms to be followed by the
countries whose scientists undertake research
studies in the North Pole and South Pole.
(d)It is a trade and security agreement of the
member countries of the Arctic Council.
Ans: B

∙ Option B is correct: The World Trade Organization’s 164 members commit to treating other members equally so they
can all benefit from each other’s lowest tariffs, highest import quotas and fewest trade barriers for goods and services. This
significant principle of non-discrimination is known as most favoured nation (MFN) treatment. There are some
exceptions, such as when members strike bilateral trade agreements or when members offer developing countries special
access to their markets.
∙ Removal of MFN status: There is no figurative procedure for suspending MFN treatment and it is not clear whether
members are obliged to inform the WTO if they do so.India pendent Pakistan’s MFN status in 2019 after a suicide attack
by a Pakistan-basedIslamist group killed 40 police. Pakistan never germane MFN status to India.
∙ Cancelling Russia’s MFN status sends a strong signal that the United States and its Western allies do not consider Russia a
economic partner in any way, but it does not in itself change conditions for trade. It does formally permit the Western allies
to increase import tariffs or impose quotas on Russian goods, or even ban them, and to restrict services out of the country.
They could also overlook Russian intellectual property rights.
Q.32) Consider the following statements with reference PYQ ASKED ON SIMILAR LINE
to Northern River Terrapin:
What is/are unique about ‘Kharai camel’, a
1. It is a species of riverine turtle native to Southeast Asia. breed found in India? [2016]
2. They are best known for their unique mass nesting 1.It is capable of swimming up to three
called Arribada. kilometres in seawater.
2. It survives by grazing on mangroves.
Which of the above statements is/are correct? 3.It lives in the wild and cannot be
domesticated.
a) 1 only Select the correct answer using the code
b) 2 only given below.
c) Both 1 and 2 a) 1 and 2 only
d) Neither 1 nor 2 b) 3 only
c) 1 and 3 only
d) 1, 2 and 3
ANS: A

∙ Statement 1 is correct: Northern River Terrapin (Batagur baska) is a species of riverine turtle native to Southeast Asia.
It is one of Asia’s largest freshwater and breakwater turtles. The species is currently found in Bangladesh and
India (in the Sunderbans), Cambodia, Indonesia and Malaysia. It is regionally extinct in Myanmar, Singapore, Thailand
and Vietnam. They spend most of their time in water and only come on land to lay eggs. They are even known to
undertake long seasonal migrations to the sandbanks where they were hatched. They have an omnivorous diet, taking
waterside plants and small animals such as clams.
∙ Statement 2 is incorrect: Olive Ridley turtles are known for their unique mass nesting, called Arribada, where
thousands of females come together on the same beach to lay eggs. Olive ridley turtles are found in warm waters of the
Pacific, Atlantic and Indian oceans. The Odisha’s Gahirmatha Marine Sanctuary is known as the world’s largest
rookery (colony of breeding animals) of sea turtles.
Q.33) Consider the following statements regarding the
Financial Action Task Force (FATF):

1. The Financial Action Task Force on Money Laundering


(FATF) was established by the G-20 Summit which was held
in New York in 1982.
2. The FATF help authorities to eliminate crime related human
trafficking and also works to stop funding for weapons of
mass destruction.
3. UAE has been placed in the Gray list.
PYQ ASKED ON SIMILAR LINE
Which of the statements given above is/are correct?

a) 1 and 2 only
b) 1 and 3 only
c) 2 and 3 only
1, 2 and 3
Ans: C

∙ Statement 1 is incorrect: The Financial Action Task Force (FATF) is the global money laundering and terrorist
financing watchdog. The inter-governmental body sets international standards that aim to inhibit these illegal activities and
the harm they cause to society. As a policy-making body, the FATF works to generate the essential political will to bring
about national legislative and regulatory reforms in these areas. It was established by the G-7 Summit that was held in
Paris in 1989. During 1991 and 1992, the FATF expanded its membership from the original 16 to 28 members. In 2000 the
FATF expanded to 31 members, and has since expanded to its current 39 members.
∙ Statement 2 is correct: FATF Standards, which ensure a co-ordinated global response to prevent organised crime,
corruption and terrorism. They help authorities go after the money of criminals dealing in illegal drugs, human
trafficking and other crimes. The FATF also works to stop funding for weapons of mass destruction.
∙ Statement 3 is correct: FATF issues two lists:- Black list (as High-Risk Jurisdictions subject to a Call for Action), Grey
list (referred to as Jurisdictions Under Increased Monitori FATF has retained Pakistan on its terrorism financing “grey
list”. Pakistan has been on the grey list since June 2018 for failing to check money laundering, leading to terror financing.
Also, UAE has been placed in the grey list in march 2022. Some of the other notable countries in the FATF gray list are
Panama and Turkey.
Q.34) With reference to the India- Japan relations, consider
the following statements:

1. India and Japan are both countries that are part of the Quad,
G-20 and G-4.
2. Japan is the largest Official Development Assistance (ODA)
partner for India.
3. Dharma Guardian, Shinyuu-Maitri, JIMEX are the bilateral
Military exercises conducted by both countries.
PYQ ASKED ON SIMILAR LINE
Which of the statements given above is/are correct?

a) 2 only
b) 1 and 3 only
c) 2 and 3 only
d) 1, 2 and 3
Ans: D

India- Japan relation:


∙ Statement 1 is correct: India is the largest democracy in the world and the seventh-largest country by area, the
second-most populous country and Japan is an island country in East Asia and Japan is the eleventh most populous
country in the world, as well as one of the most densely populated and urbanized. Both countries are Democratic countries
and have friendly relations and are part of the Quad, G-20 and G-4 and expressed their commitment to promote peace,
security, and prosperity at global level.
∙ Statement 2 is correct: Japan is the largest Official Development Assistance (ODA) partner for India. In 2011, two
countries signed a Comprehensive Economic Partnership Agreement (CEPA) to facilitate growth in bilateral trade.
During 2019-20, bilateral trade between both countries totalled US$ 11.87 billion. In 2020, India was the 18th largest
trading partner for Japan, and Japan was 12th largest trading partner for India.
∙ Statement 3 is correct: Both India and Japan engage in bilateral exercises- Dharma Guardian (Army), Shinyuu-Maitri
(Airforce), JIMEX (Navy). Since 2015, in Malabar exercises, Japan has also become a permanent participant along with
India and the US.Japan welcomes the Indo-Pacific Ocean’s Initiative (IPOI) announced in 2019 at East Asia Summit
(EAS) in Bangkok for promoting safe, secure, and stable maritime domain, sustainable use of marine resources, and
disaster prevention and management.To counter China's dominance of the supply chain in the Indo-Pacific region, both
countries launched the Supply Chain Resilience Initiative (SCRI). The Asia-Africa Growth Corridor (AAGC) between
both countries is characterized as a response to China’s Belt and Road Initiative (BRI). Etc.
Q.35) Consider the following statements with reference PYQ ASKED ON SIMILAR LINE
to Dugongs:
Q. With reference to 'dugong', a mammal found in
1. It is the only strictly herbivorous marine mammal. India, which of the following statements is/are
2. It is listed as critically endangered in the IUCN red list. correct? [2015]
3. In India, it is found exclusively only in the Gulf of 1. It is a herbivorous marine animal.
Mannar and Palk Bay region. 2. It is found along the entire coast of India.
3.It is given legal protection under Schedule I of
Which of the above statements is/are correct? the Wildlife (Protection) Act, 1972.
Select the correct answer using the codes given
a) 1 only below.
b) 3 only a) 1 and 2 only
c) 2 and 3 only b) 2 only
d) 1 and 3 only c) 1 and 3 only
d) 3 only
ANS: A

∙ Statement 1 is correct: Dugong (Dugong dugon) also called ‘Sea Cow’ is one of the four surviving species in the Order
Sirenia and it is the only existing species of herbivorous mammal that lives exclusively in the sea. Dugongs are
an important part of the marine ecosystem and their depletion will have effects all the way up the food chain.
∙ Statement 2 is incorrect: Conservation status of Dugong:
∙ IUCN Red List status: Vulnerable
∙ Wildlife Protection Act, 1972: Schedule I
∙ CITES: Appendix I.
∙ Statement 3 is incorrect: They are found in over 30 countries and in India are seen in the Gulf of Mannar, Gulf of
Kutch, Palk Bay, and the Andaman and Nicobar Islands.
Recent context: The Tamil Nadu government has decided to go ahead with the establishment of India's first
conservation reserve for the Dugong in Gulf of Mannar, Palk Bay. The reserve will be spread over an area of 500 sq.
km in Palk Bay on the southeast coast of Tamil Nadu.
Q.36) Consider the following statements: PYQ ASKED ON SIMILAR LINE
1. Intellectual property (IP) pertains to any original creation of the
human intellect such as artistic, literary, technical, or scientific
creation.
2. The Agreement on Trade-Related Investment Measures of The
World Trade Organization (WTO) is the most comprehensive
multilateral agreement on IP.
3. Earlier, India and Australia had proposed a temporary waiver on
IP rights for Covid-19 vaccines and covid related technology.

Which of the statements given above is/are correct?

a) 1 only
b) 3 only
c) 1 and 2 only
d) 1, 2 and 3
Ans: A

∙ Statement 1 is correct: Intellectual Property Rights (IPRs) pertains to any original creation of the human intellect such
as artistic, literary, technical, or scientific creation. Intellectual Property Rights (IPRs) are the rights given to persons over
the creations of their minds. They usually give the creator an exclusive right over the use of his/her creation for a certain
period of time. The protection is usually given for a finite term (typically 20 years in the case of patents).
∙ Statement 2 is incorrect:The World Trade Organization (WTO) Agreement on Trade-Related Aspects of Intellectual
Property Rights (TRIPS), which came into effect in 1995, is the most comprehensive multilateral agreement on IP.
Trade-Related Aspects of Intellectual Property Rights plays a crucial role in facilitating trade in knowledge and creativity,
in resolving trade disputes over Intellectual property lP, and in assuring World trade organization WTO members the
latitude to achieve their domestic policy objectives.
∙ Statement 3 is incorrect: Earlier, India and South Africa had proposed a temporary waiver on IP rights for Covid-19
vaccines and other Covid- related technologies at the World Trade Organization (WTO) until global herd immunity was
achieved. This proposal has come to be known as the TRIPS waiver.The waiver would enable the manufacturing of
generic versions of Covid-19 vaccines. It temporarily “removes” the protections provided by the WTO.About 100 of the
WTO’s 164 countries have supported India’s proposal, including the United States.However, to come into force, the
proposal must be approved by all of WTO’s member nations, as WTO works on consensus.
Q.37) Consider the following statements regarding Colombo
Security Conclave (CSC):

1. It helps member nations to build capacity on common security


threats.
2. Mauritius, India and Sri Lanka are the founding members and
Maldives is added as the newest member. PYQ ASKED ON SIMILAR LINE
3. It provides Maritime safety and security and Bangladesh and
Seychelles have observer status in the 5th meeting.

Which of the statements given above is/are correct?

a) 1 and 2 only
b) 1 and 3 only
c) 2 only
d) 1, 2 and 3 only
Ans: B

∙ Statement 1 is correct: Colombo Security Conclave (CSC), renamed version of National Security Advisor (NSA) Level
Trilateral security grouping on Maritime Security (formed in 2011) and it helps member nations to build capacity on
common security threats.The fifth meeting of national security advisers of the CSC identified key areas of cooperation to
enhance and strengthen regional security in the following five pillars.
o Maritime Safety and Security
o Countering Terrorism and Radicalization
o Combating Trafficking and Transnational Organized Crime
o Cyber Security, Protection of Critical Infrastructure and Technology
o Humanitarian Assistance and Disaster Relief
∙ Statement 2 is incorrect: Maldives, India and Sri Lanka are the Founding members of the Colombo security
Conclave. Mauritius is added as the newest member in the recently concluded meeting
∙ Statement 3 is correct: Bangladesh and Seychelles have observer status in the 5th meeting.
∙ CSC is seen as other step by India to enhance its maritime security. Maritime security broadly classified issues in the
maritime domain comprising national security, marine environment, economic development, and human security. CSC is
being seen as India’s outreach to the Indian Ocean to underline regional cooperation and shared security objectives.
Q.38) Consider the following statements regarding the Council of Europe:

1. It is an international organization founded in the wake of World War II to uphold human rights, democracy
and the rule of law in Europe.
2. The Council makes binding laws and has the power to enforce select international agreements reached by
European states on various topics.
3. The Council is an official United Nations Observer.

Which of the statements given above is/are correct?

a) 1 and 3 only
b) 2 only
c) 1 and 3 only
d) 1, 2 and 3
Ans: A

∙ Statement 1 is correct: Council of Europe established in 1949 by the Treaty of London and international organization
founded in the wake of World War II to uphold human rights, democracy and the rule of law in Europe. The council is
distinct from the European Union(EU).
∙ Statement 2 is incorrect: It includes 47 member states (27 of which are EU members). The Council can not make binding
laws, but it does have the power to enforce select international agreements reached by European states on various topics.
∙ Statement 3 is correct: The Council is also an official United Nations Observer. And its headquarters were established
in Strasbourg, France. The body of the Council of Europe is the European Court of Human Rights, which functions on the
basis of the European Convention on Human Rights.The council has two statutory bodies are the Committee of
Ministers, comprising the foreign ministers of each member state, and the Parliamentary Assembly, composed of members
of the national parliaments of each member state.
Q.39 Consider following statements about Volatile PYQ ASKED ON SIMILAR LINE
Organic Molecules (VOC):

1. Volatile Organic Molecules are released by petrol and


diesel vehicles, they do not originate from nature.
2. Benzene, a chemical that induces cancer, is the only
VOC included in the National ambient air quality
standards.
3. VOCs can irritate the eyes, nose and throat, damage
body organs and cause cancer.

Which of the statements given above is/are correct?

a) 1 and 2 only
b) 2 and 3 only
c) 1 and 3 only
d) 1, 2 and 3
ANS: B

∙ Statement 1 is incorrect: VOCs are carbon-containing chemicals released by petrol and diesel vehicles. However, VOCs
can have a natural origin, too.
∙ Statement 2 is correct: Human-made VOCs are a cause for concern, yet they don’t draw enough attention. Benzene, a
chemical that induces cancer, is the only VOC included in the National ambient air-quality standards. The other
pollutants under ambient air-quality standards considered are PM10, PM2.5, nitrogen dioxide, sulphur dioxide, carbon
monoxide, ozone, ammonia, lead, nickel and benzo(a)pyrene.
∙ Statement 3 is correct: VOCs can irritate the eyes, nose and throat, damage body organs and cause cancer. Long-term
exposure to VOCs is not good because the majority of the VOCs are carcinogenic (cancer-causing).
Q.40 Consider the following pairs of Protected areas and PYQ ASKED ON SIMILAR LINE
the states in which they are located:

Which of the above pairs are correctly matched?

a) 1 and 2 only
b) 1 and 4 only
c) 2 and 3 only
d) 1, 3 and 4 only
ANS: D

∙ Pair 1 is correct: Sariska Tiger Reserve is located in Aravali hills and forms a part of the Alwar District of Rajasthan.
Sariska was declared a wildlife sanctuary in 1955 and was declared the tiger reserve later in 1978, making it a part of
India's Project Tiger. The Reserve houses ruined temples, forts, pavilions and a palace. The Reserve is immensely rich in
flora and fauna, and is famous for Royal Bengal Tiger. The park has populations of leopards, Nilgai, Sambar, chital etc.
∙ Pair 2 is incorrect: The Mukundra Tiger reserve is located near Kota, Rajasthan. The Mukundra Hills was declared a
Wildlife Sanctuary in 1955 and a National Park (Mukundra Hills (Darrah) National Park) in 2004. The reserve is located
on the eastern bank of the Chambal River and is drained by its tributaries. The park is situated in a valley
formed by two parallel mountains viz. Mukundra and Gargola. The Tiger Reserve constitutes 3 Wildlife
Sanctuaries viz; Darrah, Jawahar Sagar, and Chambal.
∙ Pair 3 is correct: Nameri National Park is located in the foothills of the Eastern Himalayas in Assam. The Pakhui
(Pakke) Sanctuary of Arunachal Pradesh adjoins the Park on its North-Eastern point. It consists of semi-evergreen,
moist deciduous forests with cane and bamboo brakes and narrow strips of open grassland along rivers.
∙ Pair 4 is correct: Srivilliputhur-Megamalai Tiger Reserve is the 51st tiger reserve of the country, located in Tamil Nadu.
For creation of Srivilliputhur-Megamalai tiger reserve, the Megamalai WLS and the adjoining Srivilliputhur WLS were
clubbed together. The tiger reserve is part of the catchment area of the Vaigai river. It has a mix of tropical evergreen
forests and semi-evergreen forests, dry deciduous forests and moist mixed deciduous forests, grassland.
Q.41) Consider the following pair:

Exercise Country
1. SLINEX - Sri lanka - India
2. LAMITIYE - Seychelles - India
3. DUSTLIK - Uzbekistan – India

Which of the pairs given above is/are correctly matched?

a) 1 and 2 only
b) 2 and 3 only
c) 1 and 3 only
d) 1, 2 and 3
Ans: D

∙ Pair 1 is correct: SLINEX is a series of naval exercises between the Indian Navy and the Sri Lanka Navy. SLINEX
exercise is conducted by the Srilanka Navy. The exercise aims to enhance interoperability, improve mutual understanding
and exchange best practices and procedures for multi-faceted maritime operations between both navies. SLINEX is in
consonance with India’s policy of ‘Neighbourhood First’ and PM Modi's vision of ‘Security and Growth for All in the
Region (SAGAR)’.
∙ Pair 2 is correct: India- Seychelles Joint Military Exercise LAMITIYE Concludes at Seychelles, Exercise LAMITIYE
with Seychelles is significant in terms of growing security concerns in the Indian Ocean Region. Seychelles is an
archipelagic island country consisting of 115 islands in the Indian Ocean at the eastern edge of the Somali Sea.
∙ Pair 3 is correct: EX-DUSTLIK between the Indian army and Uzbekistan army begins at Yangiarik. The training
schedule will focus primarily on sharing tactical level drills and learning of best practices from each other. It aims at
enhancing understanding, cooperation and interoperability between two armies. The last edition of DUSTLIK was
conducted in Ranikhet (Uttarakhand) in March 2021.
Q.42) With reference to Biological and Toxin Weapons PYQ ASKED ON SIMILAR LINE
Convention BTWC, consider the following statements:

1. It is the first multilateral disarmament treaty banning an entire


category of weapons of mass
destruction.
2. In this convention, 183 States Parties and India are a party to
the convention.
3. BTWC has no implementation body but there is a review
conference mechanism every 10 years to review the
convention’s implementation.

Which of the statements given above is/are correct?

a) 1 only
b) 2 and 3 only
c) 1 and 2 only
d) 1, 2 and 3
Ans: C

∙ Statement 1 is correct: The Biological Weapons Convention (BWC) was negotiated by the Conference of the
Committee on Disarmament in Geneva, Switzerland. BWC opened for signature in 1972 and entered into force in 1975
and BWC is the first multilateral disarmament treaty banning an entire category of weapons of mass destruction (WMD).
∙ The convention effectively prevents the development, production, acquisition, transfer, stockpiling and use of biological
and toxin weapons. If a state possesses any agent, toxin, or delivery system for them, they have nine months from entry
into force of the treaty to destroy their stockpiles or divert them for peaceful use.
∙ Statement 2 is correct: In the convention 183 States Parties and 4 Signatory States and India is also a party to the
convention. Ten states have neither signed nor ratified the BTWC: Chad, Comoros, Djibouti, Eritrea, Israel, Kiribati,
Micronesia, Namibia, South Sudan, and Tuvalu.
∙ Statement 3 is incorrect: There is no implementation body of the Biological Weapons Convention (BWC) allowing
for blatant violations as seen in the past. There is a review conference every 5 years to review the convention’s
implementation and establish confidence-building measures.
Q.43) Consider the following statements regarding “hypersonic PYQ ASKED ON SIMILAR LINE
missile”:

1. A hypersonic missile is a weapon system that has five times the


speed of light and 7 times more than sound.
2. Hypersonic missiles do not follow a ballistic trajectory and can
be maneuvered to the intended target.
3. Hypersonic weapons cannot challenge detection and defense
due to their speed, manoeuvrability and high altitude of flight

Which of the statements given above is/are incorrect?

a) 1 and 3 only
b) 2 and 3 only
c) 1, 2 and 3
d) 2 only
Ans: A

∙ Statement. 1 is incorrect: A hypersonic missile is a weapon system which flies at least at the speed of Mach 5 i.e. 5 times
the speed of sound and is maneuverable. The maneuverability of the hypersonic missile is what sets it apart from a ballistic
missile as the latter follows a set course or a ballistic trajectory.
∙ Statement 2 is correct: Thus, unlike ballistic missiles, hypersonic missiles do not follow a ballistic trajectory and can be
maneuvered to the intended target. The 2 types of hypersonic weapons systems are Hypersonic Glide Vehicles (HGV) and
Hypersonic Cruise Missiles. The HGV are launched from a rocket before gliding to the intended target while the
hypersonic cruise missile is powered by air breathing high speed engines or ‘scramjets’ after acquiring their
targetAdvantages of hypersonic missilesThey can enable responsive, long range strike options against distant, defended or
time critical threatsConventional hypersonic weapons use only kinetic energy i.e. energy derived from motion, to destroy
unhardened targets or even underground facilities.
∙ Statement 3 is incorrect: Hypersonic weapons can challenge detection and defense due to their speed, maneuverability
and low altitude of flight. Ground based radars or terrestrial radars cannot detect hypersonic missiles until late in the flight
of the weapon. This delayed detection makes it difficult for the responders to the missile attack to assess their options and
to attempt to intercept the missile.
Q.44) Which of the following three countries are members of the Bucharest Nine?

1. Romania and Poland and Finland


2. Republic of Estonia, Latvia and Lithuania
3. Bulgaria, the Czech Republic and Slovakia
4. Bulgaria, Hungary and Germany
Ans: B

∙ The Bucharest Nine is a group of 9 NATO countries in Eastern Europe that became part of the US-led military alliance
after the end of the Cold War. The Bucharest Format, often abbreviated as B9, was founded on November 4, 2015.
∙ Its name comes from Bucharest, the capital of Romania. B9, it is also regarded as the “ voice of the Eastern Flank” in the
NATO alliance. The group was formed on the initiative of Klaus Iohannis, who has been President of Romania and
Andrzej Duda, who became President of Poland in August 2015.
∙ The members of Bucharest's 9 countries are Romania and Poland, Hungary, Bulgaria, the Czech Republic, Slovakia, and
the three Baltic republics of Estonia, Latvia, and Lithuania. B9 provides a platform for deepening the dialogue and
consultation among the participant allied states across the members of NATO.
∙ All 9 Nato countries were once closely aligned with the now dissolved Soviet Union, but later chose the path of
democracy. All members of the B9 are part of the European Union (EU) and NATO. The Bucharest 9 countries have been
critical of Russia’s aggression against Ukraine since 2014, when the war in the Donbas started and Russia annexed the
Crimean peninsula.
Ans: B
Q45) With reference to Tuberculosis disease, consider the
following statements:

1. Tuberculosis is caused by Mycobacterium Tuberculosis,


affecting only in the lungs of humans.
2. World Tuberculosis (TB) Day is observed every year on 24th
March.
3. India accounts for around 26% of the total TB cases across the PYQ ASKED ON SIMILAR LINE
world.

Which of the statements given above is/are correct?

a) 3 only
b) 2 and 3 only
c) 1 and 2 only
d) 1, 2 and 3
Ans: B

∙ Statement 1 is incorrect: Tuberculosis (TB) is caused by a bacterium called Mycobacterium tuberculosis, belonging to
the Mycobacteriaceae family consisting of about 200 members. TB affects the lungs of humans (pulmonary TB), but it
can also affect other organs (extra-pulmonary TB).
∙ Statement 2 is correct: World Tuberculosis (TB) Day is celebrated on 24th March to spread awareness about the
disastrous health, social, and economic consequences of Tuberculosis. It takes efforts to end the TB epidemic globally. The
theme of the year 2022 is “Invest to End TB. Save Lives.”
∙ Statement 3 is correct: According to the World health organization, in 2020, around 9,900,000 people will fall ill with TB
and die, around 1,500,000. Since the 2000 year, 66,000,000 lives have been saved by efforts taken globally to end TB.
India accounts for around 26% of the total TB cases across the world. Therefore, World TB Day is observed to educate
people around the world about the disease TB and Its impact Global Efforts:The WHO has launched a joint
initiative “Find. Treat. All. #EndTB” with the Global Fund and Stop TB Partnership.
Q.46) With reference to Man-Portable Air-Defense Systems (MANPADS), consider the following
statements:

1. MANPADS are short-range portable surface-to-air missiles fired only by helicopters and recently used by
Russia in the Ukraine war.
2. They help shield troops from aerial attacks and are used to destroy or incapacitate military tanks.
3. MANPADS have a maximum range of 8 kilometres and can engage targets at altitudes of 4.5 km.

Which of the statements given above is/are correct?

a) 2 and 3 only
b) 3 only
c) 1 and 2 only
d) 1,2and3
Ans: A

∙ Statement 1 is incorrect: The United States and North Atlantic Treaty Organization (NATO) are shipping weapons
into Ukraine, including highly sensitive items such as shoulder-fired missiles called Man-Portable Air-Defense Systems
(MANPADS) that can take down aircraft. MANPADS are short-range, lightweight and portable surface-to-air missiles that
can be fired by individuals or small groups to destroy aircraft or helicopters. Countries such as India, Pakistan, Germany,
UK,
∙ Statement 2 correct: Turkey and Israel have also used MANPADS in their defense efforts. Russia is by far the biggest
exporter of MANPADs,having sold over 10,000 such systems between 2010 and 2018 to various countries including Iraq,
Qatar, Kazakhstan, Venezuela, and Libya. They help shield troops from aerial attacks and are most effective in targeting
low-flying aircrafts. MANPADs or Man-Portable Anti-Tank Systems work in a similar manner but are used to destroy or
incapacitate military tanks.
∙ Statement 3 is correct: MANPADS have a maximum range of 8 kilometers and can engage targets at altitudes of 4.5 km.
The first MANPADS were introduced by the United States and Soviet Union in the 1960s. Most of them have passive
or ‘fire and forget’ guidance systems, meaning that the operator is not required to guide the missile to its target, enabling
them to run and relocate immediately after firing.
Q.47 Consider the following statements with reference to Halari breed of Donkey:

1. The breed is native to Saurashtra region of Gujarat.


2. These donkeys are very docile in temperament.
3. The Halari donkey’s milk has medicinal values.

Which of the above statements is/are correct?

a) 1 and 2 only
b) 2 only
c) 1 and 3 only
d) All of above
ANS: D

∙ Statement 1 is correct: Halari Donkey is native to the semi-arid landscape of Saurashtra’s Jamnagar and Dwarka
district of the state of Gujarat. These donkeys are white in colour. Forehead is mostly convex.
∙ Statement 2 is correct: These donkeys are very docile in temperament, and are used as pack animals during migration
with small ruminants by the Bharwad and Rabari communities and for transportation as donkey cart. Halary Donkey can
walk approx. 30-40 km in a day during migration. The Kumbhar (potter) community also uses this animal for pottery
work in the Jamnagar region.
∙ Statement 3 is correct: The Halari donkey’s milk is also said to be one of the most expensive in the world which can go
upto Rs 1000 for its medicinal values.
Q.48) Consider the following statements with reference to the Sagar Parikrama:

1. It is a navigation journey to be conducted in all coastal states/UTs to demonstrate solidarity with all fisherfolk,
fish farmers and concerned stakeholders.
2. It is organized by the Ministry of Fisheries, National Fisheries Development Board with the Government of
Andhra Pradesh.
3. It has been launched as a part of ‘Azadi Ka Amrit Mahotsava’.

Which of the statements given above is/are correct?

a) 1 and 2 only
b) 2 and 3 only
c) 1 and 3 only
d) 1, 2 and 3
Ans: C

∙ Statement 1 is correct: Sagar Parikrama is a navigation journey to be conducted in all coastal states/UTs to demonstrate
solidarity with all fisherfolk, fish farmers and concerned stakeholders. The navigation journey shall focus on the
sustainable balance between the utilization of marine fisheries resources for food security of the nation and livelihoods of
coastal fisher communities and protection of marine ecosystems.
∙ Statement 2 is incorrect: Sagar Parikrama is organized by the Ministry of Fisheries, National Fisheries Development
Board along with the Government of Gujarat, Indian Coast Guard, Fishery Survey of India and Gujarat Maritime Board. It
will start from Mandvi (Gujarat) in its first phase and will be organized in other districts of Gujarat and other States/UTs in
the subsequent phase.
∙ Statement 3 is correct: Sagar Parikrama has been launched as a part of ‘Azadi Ka Amrit Mahotsava’ saluting our great
freedom fighters, sailors and fishermen. Azadi Ka Amrit Mahotsava is one of the significant initiatives of the Government
of India to celebrate and commemorate 75 years of independence and the glorious history of its people, culture and
achievements.
Q.49 Africa’s Boma technique was in news recently in PYQ ASKED ON SIMILAR LINE
the context of:
Q. The term ‘M-STrIPES’ is sometimes seen in the
a) Growing trees on degraded land. news in the context of [2017]
b) Capturing and transporting wild animals. a) Captive breeding of Wild Fauna
c) Water conservation in the desert areas. b) Maintenance of Tiger Reserves
d) None of the above c) Indigenous Satellite Navigation System
d) Security of National Highways
ANS: B

∙ About Africa’s Boma Technique:


∙ The Boma capturing technique is popular in Africa.
∙ It involves luring animals into an enclosure by chasing them through a funnel-like fencing.
∙ The funnel tapers into an animal selection-cum-loading chute, supported with grass mats and green net to make it opaque
for animals, which are herded into a large vehicle for their transport to another location.
∙ This old technique was earlier utilized to capture wild elephants for training and service.
∙ Following its adoption in Madhya Pradesh in recent years, Boma has been put to practice for the first time in Rajasthan for
sending the ungulates to the prey-deficient Mukundara reserve as the kills for tigers and leopards.
∙ This shifting exercise has been approved by National Tiger Conservation Authority’s (NTCA).
Q.50) Consider the following statements with reference to the
Katchatheevu Island:

1. It is an uninhabited islet in the Palk Strait, formed due to


volcanic eruption.
2. Prime Minister of India, Rajeev Gandhi ceded Katchatheevu
island to Sri Lanka under the "Indo-Sri Lankan Maritime
agreement".
PYQ ASKED ON SIMILAR LINE
Which of the statements given above is/are correct?

a) 1 only
b) 2 only
c) Both 1 and 2
d) Neither 1 nor 2
ANS: A

∙ Statement 1 is correct: Katchatheevu Island is an uninhabited islet in the Palk Strait, formed due to volcanic eruption in
the 14th century. It was administered jointly by India and Sri Lanka during British rule. It is strategically important for
fishing activities.
∙ Statement 2 is incorrect: Both Sri Lanka and India claimed this island for fishing and the dispute remained unsettled till
1974. In 1974, then Prime Minister of India, Indira Gandhi ceded Katchatheevu to Sri Lanka under the "Indo-Sri Lankan
Maritime agreement". Another agreement signed in 1976 restricted both the countries’ fishermen from fishing in the
other’s exclusive economic zones.
ANS: A
Q.51. Which of the following statements is/are correct regarding the PM- Shram Yogi Maan-Dhan
Scheme?

1. It is a 50:50 voluntary and contributory pension scheme in which the beneficiary makes a stipulated
age-specific contribution and the Central Government matches it.
2. Unorganised sector workers in the age group of 18-40 years will be eligible for the scheme.
3. After attaining the age of 60 years, registered workers will receive the minimum assured pension of Rs 5000
per month.

Select the correct answer using the code given below:

a) 1 and 2 only
b) 2 and 3 only
c) 1 and 3 only
1, 2 and 3
ANS: A

∙ Statement 1 is correct: PM- Shram Yogi Maan-Dhan Scheme is a 50:50 voluntary and contributory pension scheme in
which the beneficiary makes a stipulated age-specific contribution and the Central Government matches it. The Ministry of
Labour and Employment will oversee PM-SYM, which will be implemented by Life Insurance Corporation of India and
CSC eGovernance Services India Limited.
∙ Statement 2 is correct: Workers working in the unorganized sector in the age group of 18-40 years will be eligible and
can register themselves and deposit a minimum amount of Rs 660 to 2400 every year depending on their age. They
should not be covered under the New Pension Scheme (NPS), Employees’ State Insurance Corporation (ESIC) scheme or
Employees’ Provident Fund Organization (EPFO). He/She should not be an income tax payer.
∙ Statement 3 is incorrect: After attaining the age of 60 years, each registered worker shall receive the minimum assured
pension of Rs 3,000 per month. This scheme seeks to benefit around 42 crore unorganized sector workers of the country.
Q.52) Which one of the following ministries is the parent PYQ ASKED ON SIMILAR LINE
organization of Agricultural and Processed Food Products
Export Development Authority (APEDA)?

a) Ministry of Agriculture and Farmers Welfare


b) Ministry of Food Processing Industries
c) Ministry of Consumer Affairs, Food and Public Distribution
d) Ministry of Commerce and Industry
ANS: D

∙ Option D is correct: Agricultural and Processed Food Products Export Development Authority (APEDA) was set-up
by the Government of India under the Agricultural and Processed Food Products Export Development Authority Act, 1985.
APEDA promotes exports of agricultural commodities and fresh and processed products by setting standards and
specifications. It has been entrusted with the responsibility to monitor the import of sugar. The Ministry of Commerce and
Industry is the parent organization of APEDA. The Headquarters of APEDA is in New Delhi.
Q.53) Consider the following statements with reference to the Start-Up Village Entrepreneurship
Programme (SVEP):

1. It is a sub-scheme under the Deendayal Antyodaya Yojana-National Rural Livelihoods Mission.


2. It promotes only group enterprises set-up and promotes enterprises majorly in agriculture sectors.

Which of the statements given above is/are incorrect?

a) 1 only
b) 2 only
c) Both 1 and 2
Neither 1 nor 2
ANS: B

∙ Statement 1 is correct: Start-Up Village Entrepreneurship Programme (SVEP) was launched in 2016 by the Ministry
of Rural Development as a sub-scheme under the Deendayal Antyodaya Yojana – National Rural Livelihoods Mission.
Deendayal Antyodaya Yojana-National Rural Livelihoods Mission is a centrally sponsored programme with the aim to
eliminate rural poverty through the promotion of multiple livelihoods and improved access to financial services for the
rural poor households across the country.
∙ Statement 2 is incorrect: Start-Up Village Entrepreneurship Programme (SVEP) promotes both individual and group
enterprises set-up and promotes enterprises majorly in manufacturing, trading and service sectors . Entrepreneurship
Development Institute of India (EDII), Ahmedabad is the technical support partner of SVEP.
Q.54) Which of the following statements is/are correct PYQ ASKED ON SIMILAR LINE
regarding the UPI123Pay?

1. It is a two-step method to initiate and execute services for


users.
2. It will work on simple phones and doesn’t need an internet
connection for transactions.

Select the correct answer using the code given below:

a) 1 only
b) 2 only
c) Both 1 and 2
d) Neither 1 nor 2
ANS: B

∙ Statement 1 is incorrect: Reserve Bank of India launched a new Unified Payments Interface (UPI) payments solution for
feature phone users dubbed ‘UPI123Pay’ and it is a three-step method to initiate and execute services for users which will
work on simple phones.
∙ Statement 2 is correct: UPI123Pay will allow customers to use feature phones for almost all transactions except scan and
pay and doesn’t need an internet connection for transactions. Customers will have to link their bank account with feature
phones to use this facility. The new UPI payments system offers users four options to make payments i.e. Interactive Voice
Response; App-based functionality; Missed call facility and Proximity sound-based payments.
Q.55) Which one of the following states has topped the Niti Aayog’s Export Preparedness
Index, 2021?

a) Maharashtra
b) Gujarat
c) Telangana
d) Karnataka
ANS: B

∙ Option B is correct: Export Preparedness Index, 2021 is released by NITI Aayog in collaboration with the Institute of
Competitiveness. It is a comprehensive analysis of India’s export achievements. It aims to identify the fundamental areas
critical for subnational export promotion. The coastal states have been adjudged as the best performers in the
index. Gujarat has topped NITI Aayog’s Export Preparedness Index (EPI) 2021, Maharashtra has been ranked second
and Karnataka has been ranked third.
Q.56) Consider the following statements with reference to the Pradhan Mantri Garib Kalyan Anna
Yojana:

1. It was launched by the Department of Food and Public Distribution under the Ministry of Consumer Affairs,
Food and Public Distribution.
2. It is aimed to ensure sufficient food for poor and needy people during the coronavirus crisis.
3. It would be funded by the Central and State Government both.

Which of the statements given above is/are correct?

a) 1 and 2 only
b) 2 and 3 only
c) 1 and 3 only
d) 1, 2 and 3
ANS: A

∙ Statement 1 is correct: Pradhan Mantri Garib Kalyan Anna Yojana was launched by the Department of Food and Public
Distribution under the Ministry of Consumer Affairs, Food and Public Distribution in April 2020. The union cabinet has
extended the PM-GKAY scheme for another six months till September 2022. This will be the sixth phase of PM-GKAY.
∙ Statement 2 is correct: Pradhan Mantri Garib Kalyan Anna Yojana is aimed to ensure sufficient food for poor and needy
people during the coronavirus crisis. Under the scheme, each beneficiary will get an additional 5 kg free ration per
person per month in addition to his normal quota of foodgrains under the National Food Security Act (NFSA).This means
that every poor household would get nearly double the normal quantity of ration.
∙ Statement 3 is incorrect: The scheme will cover nearly 80 crore beneficiaries across India and would be fully funded by
the Government of India. The benefit of the free ration can also be availed through portability by any migrant labor or
beneficiary under the One Nation One Ration Card (ONORC) plan from nearly 5 lakh ration shops across the country.
Q.57) Which of the following statements is/are correct PYQ ASKED ON SIMILAR LINE
regarding the Zoji La Pass?

1. It connects Leh and Srinagar and provides important links


between Union Territories of Ladakh and Kashmir.
2. It is known as the “Mountain Pass of Blizzards”.
3. It is a high mountain pass located in Leh district of Ladakh.

Select the correct answer using the code given below:

a) 1 and 2 only
b) 2 and 3 only
c) 1 and 3 only
d) 1, 2 and 3
ANS: A

∙ Statement 1 is correct: Zoji La Pass connects Leh and Srinagar and provides important links between Union Territories
of Ladakh and Kashmir. The Zoji La Tunnel Project was launched in 2018. It is Asia's longest and strategic
bi-directional tunnel, which will provide all weather connectivity between Srinagar, Kargil and Leh.
∙ Statement 2 is correct: Zoji La Pass is known as the “Mountain Pass of Blizzards”. It remains closed during winters due
to heavy snowfall, cutting off Ladakh region from Kashmir. The Vijayak Force unit and the Beacon Force Unit of the
Border Road Organization (BRO) are responsible for the maintenance and clearing of the road during winter.
∙ Statement 3 is incorrect: Zoji La Pass is a high mountain pass located in Kargil district of Ladakh. Pakistani raiders seized
this pass during the Indo-Pakistani war of 1947-48 as part of their campaign to capture Ladakh. The Indian forces
recaptured this pass in an assault codenamed Operation Bison.
Q.58) Consider the following statements regarding the Karewa PYQ ASKED ON SIMILAR LINE
Sediments:

1. It means “elevated table land” in kashmiri dialect.


2. It is ideal for the cultivation of saffron, almonds, apples and
several other cash crops.
3. It is found primarily in the Valley of Kashmir and in
Bhaderwah Valley of the Jammu Division.

Which of the statements given above is/are correct?

a) 1 and 2 only
b) 2 and 3 only
c) 1 and 3 only
d) 1, 2 and 3
ANS: D

∙ Statement 1 is correct: : Karewas are lacustrine deposits (deposits in lake). It is an intermountain valley fill, consisting
of thick deposits of glacial clay and other materials embedded with moraines. Karewa means “elevated table land” in
kashmiri dialect.
∙ Statement 2 is correct: Karewas are 13,000-18,000-meter-thick deposits of alluvial soil and sediments like sandstone
and mudstone. This makes them ideal for the cultivation of saffron, almonds, apples and several other cash
crops. Kashmiri saffron received a Geographical Indication (GI) tag in 2020 for its longer and thicker stigmas, deep-red
colour, high aroma and bitter flavour.
∙ Statement 3 is correct: Karewas are found primarily in the Valley of Kashmir and in Bhaderwah Valley of the Jammu
Division. “Vudr” is the local name for Karewas in Kashmiri language.
Q.59) Consider the following statements with reference to the
Asani Cyclone:

1. It is formed over the Arabian Sea and hits the coastal region of
Gujarat and Maharashtra.
2. The name of the cyclone ‘Asani’, has been suggested by Oman.

Which of the statements given above is/are correct?


PYQ ASKED ON SIMILAR LINE
a) 1 only
b) 2 only
c) Both 1 and 2
d) Neither 1 nor 2
ANS: D

∙ Statement 1 is incorrect: Asani, the first cyclone to hit India in 2022, is formed over the southeast Bay of Bengal. It is
expected to travel along and off the coast of the Andaman and Nicobar Islands before it intensifies into a
depression. A Cyclone is a large-scale air mass that rotates anti-clockwise in northern hemisphere and clockwise in the
southern hemisphere around a strong center of low atmospheric pressure. Wet and warm air at the surface of the ocean
rises higher. This causes a zone of low pressure near the surface. Due to this, cold air from the surrounding areas flows into
the area of low pressure which causes the cold air to turn warm and wet causing it to rise. The cycle continues which
results in cloud development. This wind and cloud system then expands as well as rotates which culminates into a cyclone.
∙ Statement 2 is incorrect: The name of the cyclone ‘Asani’, has been suggested by Srilanka “Ashani” translates to
“Wrath” in the Sinhala language.. There are six regional meteorological centres across the world with the mandate of
naming tropical cyclones and the Indian Meteorological Department is one of them. New Delhi is mandated to name
tropical cyclones that develop over the north Indian Ocean including the Bay of Bengal and the Arabian Sea.
∙ The names and lists of cyclones are maintained by the World Meteorological Organization’s (WMO) international
council. They also update this list of names. In the original list, only female names were added. In 1979, male names were
added to be used for the naming of cyclones. The naming lists are used alternatively. In a rotating fashion, six different lists
are being utilised to name the cyclones. Names are proposed by thirteen member countries - Bangladesh, India,
Maldives, Myanmar, Oman, Pakistan, Sri Lanka, Thailand, Iran, Qatar, Saudi Arabia, United Arab Emirates and Yemen.
Q.60) Which of the following statements is/are correct
regarding the Deep Ocean Mission?

1. It aims to explore the deep oceans for resources and the


technology and expertise needed in such missions are available
in only five countries - the US, Russia, France, Japan and
China. India will now be the sixth country to have it.
2. The Ministry of Fisheries, Animal Husbandry and Dairying PYQ ASKED ON SIMILAR LINE
will be the nodal ministry for implementing this mission.

Select the correct answer using the code given below:

a) 1 only
b) 2 only
c) Both 1 and 2
Neither 1 nor 2
ANS: A

∙ Statement 1 is correct: Deep Ocean Mission aims to explore the deep ocean for resources, sustainable use of ocean
resources and develop deep-sea technologies. It will be a mission mode project to support the Blue Economy Initiatives of
the Government of India. Blue Economy is the sustainable use of ocean resources for economic growth, improved ocean
ecosystem health and better livelihoods and jobs. Three sides of India are surrounded by the oceans and around 30% of the
country's population lives in coastal areas, the ocean is a major economic factor supporting fisheries and aquaculture,
tourism, livelihoods and blue trade. The technology and expertise needed in such missions are now available in only five
countries - the US, Russia, France, Japan and China. India will now be the sixth country to have it.
∙ Statement 2 is incorrect: Ministry of Earth Sciences(MoES) will be the nodal Ministry for implementing the Deep
Ocean Mission. India has a unique maritime position, having 7517 km of coastline and 1382 islands. Considering the
importance of the oceans on sustainability, the United Nation has declared the 2021-2030 as the decade of Ocean Science
for Sustainable Development.
Q.61) Consider the following statements about the P-8Is aircraft:

1. It is a long-range maritime patrol aircraft.


2. They had been deployed for extensive search and rescue operations in the aftermath of Cyclone ‘Tauktae’.
3. This 'Game Changer' aircraft is the platform of choice for detecting and neutralizing enemy ships and
submarines in the Indian Ocean Region.

Which of the statements given above is/are correct?

a) 1 and 2 only
b) 3 only
c) 2 and 3 only
d) 1, 2 and 3
ANS: D

∙ Statement 1 is correct: It is a long-range maritime patrol aircraft. The Indian Navy procured four P-8I aircraft as part of
the optional clause in 2016 in a deal worth over $1 bn, deliveries of which were completed recently by Boeing. It will
significantly enhance squadron ability to protect, preserve and promote India’s maritime interests,” The squadron has been
named ‘Condors’, one of the largest flying land birds with a massive wingspan.
∙ Statement 2 is correct: They had been deployed for extensive search and rescue operations in the aftermath of Cyclone
‘Tauktae’, and rendered Humanitarian Assistance and Disaster Relief (HADR) assistance post-cyclone
‘Hudhud’. The aircraft were also deployed for support to friendly foreign countries during the COVID-19 pandemic as
part of the government’s ‘Vaccine Maitri’ initiative and participated with distinction in numerous multinational exercises
∙ Statement 3 is correct: It is a potent platform for maritime surveillance and strike, electronic warfare missions, search and
rescue, providing targeting data to weapon platforms, time-critical surveillance information to the Army and the Air
Force. INAS 316 will operate the Boeing P-8I aircraft, a multi-role Long Range Maritime Reconnaissance Anti-Submarine
Warfare (LRMR ASW) aircraft, that can be equipped with a range of Air-to-Ship Missiles and Torpedoes. The 'Game
Changer' aircraft is a potent platform for Maritime Surveillance and strike, Electronic warfare missions, Search and
Rescue, providing targeting data to Weapon platforms, providing time-critical surveillance information for IA and IAF and
is also the platform of choice for detecting and neutralising enemy ships and submarines in Indian Ocean Region.. It was
also the platform of choice for detecting and neutralizing enemy ships and submarines in the IOR.
∙ Agreement with U.S.: The six P-8Is will come installed with encrypted communication systems as India has now signed
the foundational agreement Communications Compatibility and Security Agreement (COMCASA) with the U.S.
Q.62) Consider the following statements about the Param Shakti:

1. It is not a Petascale Supercomputer.


2. It is under the National Supercomputing Mission (NSM).
3. It provides large-scale computing power to the user community.

Which of the statements given above is/are correct?

a) 1 and 2 only
b) 3 only
c) 2 and 3 only
d) 1, 2 and 3
ANS: D

∙ Statement 1 is incorrect: Param Shakti is a Petascale Supercomputer that was inaugurated at IIT Kharagpur.
∙ Statement 2 is correct: PARAM Shakti has been dedicated to the nation under the National Supercomputing Mission
(NSM). NSM is a joint initiative of the Ministry of Electronics and Information Technology (MeitY) and the Department
of Science and Technology (DST).
∙ Statement 3 is correct: Benefits of PARAM Shakti supercomputing facility are:
∙ accelerating the research and development activities in multidisciplinary domains of computational and data
sciences provides large-scale computing power to the user community would aid researchers to solve large-scale problems
in different fields ofScience and Engineeringmajor boost to the research and development initiatives in Indian academia
and industries to reach a position of global esteem.
Q.63 Consider the following difference between African and Asiatic Cheetah:

1. African Cheetahs are bigger than Asiatic Cheetahs with sturdier legs and necks.
2. African Cheetah has light brown colour fur, whereas Asiatic Cheetah has pale yellow colour fur.
3. African cheetahs are found in much larger areas than Asiatic Cheetahs.

Which of the above statements are correct?

a) 1 and 2 only
b) 2 and 3 only
c) 1 and 3 only
d) 1, 2 and 3
ANS: D

∙ Statement 1 is correct: The Asiatic cheetah is slightly smaller and slender than the African cheetah. The neck is
much smaller and longer. Their legs are also slender. African cheetahs, on the other hand, have a slightly bigger
build with sturdier legs and necks. Their heads are also bigger compared side by side with the Asiatic cheetahs.
∙ Statement 2 is correct: The Asiatic cheetah has a buff to light fawn color bordering on pale yellow skin, and it has more
fur on the body, especially under the belly and the back of the neck. The African cheetah has a light brown to golden
brown fur color that is thicker than the Asiatic ones. The spots are more pronounced on the face and are more densely
distributed on the body compared to Asiatic cheetahs.
∙ Statement 3 is correct: The African cheetah is spread out across Africa from Northwest Africa, East Africa, and
Southern Africa. With a bigger territory, the African cheetahs have the highest populations compared to their Asiatic
counterparts. On the other hand, Asiatic Cheetahs are only found in a small region between Iran and Pakistan.
Q.64) Consider the following statements about the Indian Army version of Medium Range Surface to Air
Missile (MRSAM):

1. It is jointly developed by DRDO and Israel Aerospace Industries (IAI), Israel.


2. The missiles intercepts the aerial targets but cannot destroy them completely
3. The MRSAM Army weapon system consists of a multi-function radar and mobile launcher system.

Which of the statements given above is/are correct?

a) 1 and 2 only
b) 1 and 3 only
c) 2 and 3 only
d) 1, 2 and 3
ANS: B

∙ Statement 1 is correct: The Defense Research and Development Organization (DRDO) conducts a successful flight test
of the Indian Army version of Medium Range Surface to Air Missile (MRSAM) at Integrated Test Range, Chandipur off
the coast of Odisha. This version of MRSAM is a Surface-to-Air Missile jointly developed by DRDO and Israel
Aerospace Industries (IAI), Israel for use by the Indian Army.
∙ Statement 2 is incorrect: The missiles intercepted the aerial targets and destroyed them completely and registered high
precision. These successful tests are considered major milestones for realization of the vision of ‘Atma Nirbhar
Bharat’.
∙ Statement 3 is correct: The MRSAM Army weapon system consists of multi-function radar, mobile launcher system and
other vehicles. The tests were carried out as part of the live firing trials against high-speed aerial targets.
Q.65) Consider the following statements about the Tejas skilling project :

1. It was launched at the Dubai Expo.


2. It is a Skill India International Project to train Indians abroad.
3. The project is aimed to increase the skills, certifications and overseas employment opportunities for Indians.

Which of the statements given above is/are correct?

a) 1 and 2 only
b) 1 and 3 only
c) 2 and 3 only
d) 1, 2 and 3
ANS: D

∙ Statement 1 is correct: This programme has the objective of skilling, providing certification, and employment of Indians
overseas. TEJAS is also aiming to create pathways so that the Indian workforce can get equipped for the various skills and
market requirements in the UAE. Under this project, 10,000 strong Indian workforces will be created in the UAE during
the initial phase of implementation. TEJAS skilling project was launched by the union minister at the Dubai Expo.
∙ Statement 2 is correct: TEJAS (Training for Emirates Jobs And Skills) is a Skill India International Project to train
Indians abroad. This project will be looking to skill the Indian population overseas thus providing the world with a large
workforce from India who is skilled. TEJAS project will create a pathway between UAE and India. Through this project’s
implementation pathways will be created between India and the UAE enabling the Indian workforce according to the needs
of the UAE market.
∙ Statement 3 is correct: The project is aimed to increase the skills, certifications and overseas employment opportunities
for Indians. The main objective of the project is to create pathways to enable the Indian workforce to get equipped for
skill and market requirements in UAE.
Q.66) Consider the following statements about the High Altitude Platforms (HAP):

1. It is a solar-powered UAV capable of day & night operation.


2. It operates at a height of 50 km for more than 150 day.
3. It will be used for broadband communication, surveillance, earth observation and climate research.

Which of the statements given above is/are correct?

a) 1 and 2 only
b) 1 and 3 only
c) 2 and 3 only
d) 1, 2 and 3
ANS: B

∙ Statement 1 is correct: NAL demonstrated a functional model of High Altitude Platforms (HAP) which is based on
futuristic path-breaking technology. HAP is a solar-powered UAV capable of day & night operation.
∙ Statement 2 is incorrect: HAP operates at a height of 20 km for more than 90 days. HAP is regarded as a
game-changer in the fields of pseudo satellites for telecommunication applications in the 5G & 6G spectrum.
∙ Statement 3 is correct: HAP will be used for various purposes such as broadband communication, surveillance, earth
observation, climate research, etc.
Q.67) Consider the following statements about the Kavach Technology:

1. It has been developed by the Research Designs & Standards Organization (RDSO).
2. It is an indigenously developed Automatic Train Protection (ATP) System.
3. It will aid Loco Pilots to avoid Signal Passing At Danger (SPAD) and over speeding.
4. It enables auto whistling at Level Crossing gates.

Which of the statements given above is/are correct?

a) 1 and 2 only
b) 2 and 3 only
c) 1, 2 and 4 only
1, 2, 3 and 4
ANS: D

∙ Statement 1 is correct: KAVACH has been developed by the Research Designs & Standards Organization (RDSO) along
with three Indian vendors.
∙ Statement 2 is correct: Indian Railway has developed indigenous Automatic Train Protection (ATP) System called
Kavach. The technology is aimed at enhancing safety and efficiency of train operations.
∙ Statement 3 is correct: The technology will aid Loco Pilots to avoid Signal Passing At Danger (SPAD) and over speeding
and also help in operations during extreme weather conditions.
∙ Statement 4 is correct: Key Features
∙ It controls the speed of the train by automatic application of brakes.
∙ It is a set of electronic devices and Radio Frequency Identification (RFID) devices installed in locomotives, in the
signaling system as well as the tracks.
∙ It enables auto whistling at Level Crossing gates.
∙ It helps in preventing collisions by facilitating loco-to-loco communication.
∙ It also supports features such as emergency response mechanisms in case of any mishap.
Q.68) Consider the following statements with reference to the
Konark Temple:

1. The temple is a World Heritage Site and dedicated to the Hindu


God Sun.
2. It was built by King Narasimhadeva I of the Eastern Ganga
dynasty.
3. It is depicted on the reverse side of the Indian Currency note of
20 rupees.
PYQ ASKED ON SIMILAR LINE
Which of the statements given above is/are correct?

a) 1 and 2 only
b) 2 and 3 only
c) 1 and 3 only
d) 1, 2 and 3
ANS: A

∙ Statement 1 is correct: Konark Temple is a World Heritage Site and dedicated to the Hindu God Sun. It is situated in the
eastern state of Odisha. It forms part of the golden triangle of Odisha, along with Puri and Bhubaneswar. The Sun temple
belongs to the Kalinga School of Indian Temples which consists Shikhara (crown), Jagmohana (audience hall),
Natmandir (dance hall), Vimana (tower).
∙ Statement 2 is correct: Konark Sun Temple was built by King Narasimhadeva I of the Eastern Ganga dynasty from
1238-1250 CE. The temple was called by European sailors "The Black Pagoda" as it formed an important landmark for
them in their coastal voyage.
∙ Statement 3 is incorrect: Konark Temple is depicted on the reverse side of the Indian Currency note of 10 rupees and
Ellora Caves is depicted on the reverse side of the note of 20 rupees.
Q.69) Which of the following statements is/are correct
regarding the Perini Dance?

1. It is an ancient dance form, originated and prospered in


Telangana during the Kakatiya dynasty.
2. It is a dance form usually performed by females.
3. It is performed in honour of Lord Siva, the hindu god of
destruction.
PYQ ASKED ON SIMILAR LINE
Select the correct answer using the code given below:

a) 1 and 2 only
b) 2 and 3 only
c) 1 and 3 only
d) 1, 2 and 3
ANS: C

∙ Statement 1 is correct: Perini Dance is an ancient dance form, originated and prospered in Telangana during the Kakatiya
dynasty. The Kakatiya Dynasty ruled most of the eastern deccan region comprising present day Telangana and Andhra
Pradesh and parts of eastern Karnataka and southern Odisha between 12th and 14th centuries. Orugallu (Warangal) is the
capital of Kakatiya dynasty.
∙ Statement 2 is incorrect: Perini Dance is a dance form usually performed by males. The evidence of this dance can be
seen in the sculptures near Garbha Gudi (Sanctum Sanctorum) of the Ramappa Temple at Warangal. Ramappa Temple is a
UNESCO World Heritage Site.
∙ Statement 3 is correct: Perini Dance is performed in honour of Lord Siva, the hindu god of destruction. It is called ‘Dance
of Warriors’. Warriors before leaving to the battlefield enact this dance before the idol of Lord Siva.
Q.70) Which of the following statements is/are correct PYQ ASKED ON SIMILAR LINE
regarding the festival Navroz in Medieval India?

1. Iltutmish introduced the famous Persian festival Navroz


in India.
2. Aurangzeb abolished the nine-day festival of Navroz.

Select the correct answer using the code given below:

a) 1 only
b) 2 only
c) Both 1 and 2
d) Neither 1 nor 2
ANS: B

∙ Statement 1 is incorrect: Navroz is the new year celebrations for Parsis (Zoroastrians) and Muslims (both Shia and
Sunni). In 1079 AD, a Persian (Iranian) king named Jalaluddin Malekshah introduced this festival to generate revenue
and collect taxes from people. Balban introduced the famous Persian festival Navroz in India.
∙ Statement 2 is correct: Aurangzeb abolished the festival of Navroz, as it was considered as Zoroastrian practice favored
by the Safavid rulers of Iran. Navroz is inscribed in the list of UNESCO Intangible Cultural Heritage of Humanity of
India.
Q.71) In which Indian states, the Karakattam dance is PYQ ASKED ON SIMILAR LINE
performed in praise of the rain goddess Mariamman?

a) Tamilnadu and Karnataka


b) Tamilnadu and Kerala
c) Karnataka and Kerala
d) Karnataka and Telangana
ANS: B

∙ Option B is correct: Karakattam is an ancient folk dance, performed in praise of the rain goddess Mariamman. It
is performed in different parts of Tamilnadu and Kerala. It involves balancing a large number of pots of decreasing
size on the head of the dancers while dancing and expressing the theme of the dance. Recently, Kerala Nattukala Kshema
Sabha has demanded that Karakattam (known as Kumbakkali in certain parts) be recognised as an agricultural art form of
Kerala.
Q.72) Which of the following statements is/are correct regarding the Vinayak Damodar Savarkar?

1. He was against foreign goods and propagated the idea of Swadeshi.


2. He worked on the abolishment of untouchability.
3. He was the founder of Hindu Mahasabha.

Select the correct answer using the code given below:

a) 2 and 3 only
b) 1 and 3 only
c) 1 and 2 only
d) 1, 2 and 3
ANS: C

∙ Statement 1 is correct: Vinayak Damodar Savarkar was an Indian politician, activist and writer. He developed the Hindu
nationalist political ideology of Hindutva. He was against foreign goods and propagated the idea of Swadeshi. He formed a
youth organization Mitra Mela. This organization was put into place to bring in national and revolutionary ideas.
∙ Statement 2 is correct: Savarkar championed atheism and rationality and also disapproved orthodox Hindu belief. In fact,
he even dismissed cow worship as superstitious. He also Worked on abolishment of untouchability in Ratnagiri. Dr
Babasaheb Ambedkar also compared his work to Lord Buddha.
∙ Statement 3 is incorrect: Savarkar was a leading figure in the Hindu Mahasabha but he is not the founder of Hindu
Mahasabha. Hindu Mahasabha was founded in 1915 by Madan Mohan Malviya to protect the rights of the hindu
community in British India. The first All India Hindu Mahasabha conference was organized at Haridwar in 1915.
Q.73) Which one of the following was known as the “First Muslim woman teacher of
India”?

a) Aruna Aasaf Ali


b) Abadi Banu Begum
c) Fatima Begum Sheik
d) Jahanara Shahnawaz
ANS: C

• Option C is correct: Fatima Begum Sheik was an Indian educator and social reformer. She was a colleague of the social
reformers Jyotirao Phule and Savitribai Phule. She is widely considered as the India’s first Muslim Woman Teacher. She
was the sister of Mian Usman Sheik, in whose house Jyotirao and Savitribai Phule took up residence.

You might also like